You are on page 1of 79

For

L ectu rers

A p ril 2 01 3
Chief Advisor
Anita Satia
Director, SCERT

Guidance
Dr. Pratibha Sharma,
Joint Director, SCERT

Academic Co-ordinator and Editor


Sapna Yadav, Sr. Lecturer, SCERT

Contributors
C.B. Verma Retd. Principal
Pundrikaksh Kaundinya, Principal, Co.Ed. G. SSS, Sector 15, Rohini
Sher Singh, Principal, Navyug School, Lodhi Road
Devendra Kumar, Lecturer, RPVV, Civil Lines
Girija Shankar, Lecturer, SKV, Yamuna Vihar
Chitra Goel, Retd Vice Principal
Krishan Dev Sharma, Lecturer, RPVV, Raj Niwas Marg
Dr.Rajesh Kumar, Principal, DIET Daryaganj
Sapna Yadav, Sr. Lecturer , SCERT

Publication Officer
Ms. Sapna Yadav
Ms. Meenakshi Yadav

Publication Team
Sh. Navin Kumar, Ms. Radha, Sh. Jai Baghwan

Published by : State Council of Educational Research & Training, New Delhi and printed at
Educational Stores, S-5, Bsr. Road Ind. Area, Ghaziabad (U.P.)
Editorial

Physics is the general analysis of nature, conducted in order to understand how the universe behaves.
The universe is commonly defined as the totality of everything that exists or is known to exist.
Physics is a part of natural philosophy and a natural science that involves the study of matter and
its motion through space and time, along with related concepts such as energy and force. By studying
physics one can understand how scientific theories explain natural phenomena and how scientific
laws identify regularities and patterns in nature, as new evidence emerges, and/or new ideas and
interpretations are advanced.

The students may or may not continue to study physics beyond the higher secondary stage, but we
feel that the thought process of physics useful in any other branch they may like to pursuer, be it
finance Administration , Social Science, Environment, Engineering, Technology, Biology or Medicine.

The present manual has been designed keeping in view the needs of the teachers as they face
challenges in transacting the curriculum in their classroom. This manual has been prepared for
support of the professional development of In-Service Teachers of Physics. This manual has contains
the Learning Objectives of some important chapters so that teachers can go through those concepts
before entering into the classroom. Also gives the Activities of some concepts for supporting the
activity based learning and to develop skills and acquire values for positive attitude in life. It must
be remembered that entire physics is based on observation & experiments without which a theory
does not get acceptance in to the domain of physics. This manual is designed for teachers keeping
in mind the theory and practices and the present examination system which is meant for testing the
problems from different topics. The designing of question in this manual is according to the trend
being adopted by the examining body. The difficulty level has been raised gradually, so that all the
teachers gets benefited equally, and accordingly. We can also observe very interesting experiments
on various topics of physics on youtube.com.

We are thankful to all those who conveyed these inputs.

We welcome suggestions and comments from our valued users, especially students and teachers at
the email id scert.physics@gmail.com & can also post on www.scertdelhi.nic.in.

We wish our readers a happy journey to the exciting realm of physics.


Contents

S.No. Name Page No.

Editorial (iii)
Introducing Life Skills and Values in Teaching of Physics (vi)
Tips to Prepare Physics students for CBSE Board Examination (viii)
Activities to Create Interest in the Subject (xviii)
Problem Solving Assessment (PSA) for Class XI (xxiv)
Physics Syllabus (xxxvii)

1. Heat and Thermodynamics 01-10

2. Oscillations and Waves 11-33

3. Electrostatics 34-61

4. Current Electricity 62-76

5. Electromagnetic Induction and Alternating Current 77-83

6. Optics 84-94

7. Electronics Devices 95-109

Appendix 110-116

(iv)
Safety Precautions for Students in Physics Laboratory
● Designing of all science laboratories according to necessary norms and standards.
● Two wide doors for unobstructed exits from the laboratory.
● Adequate number of fire extinguishers near science laboratories.
● Periodical checking of vulnerable points in the laboratories in relation to possibility of any
mishappening.
● Periodical checking of electrical fittings/ insulations for replacement and repairs.
● Timely and repeated instructions to students for careful handling of equipments in the laboratory.
● Display of do's and dont's in the laboratory at prominent places.
● Safe and secure storage of all Equipments
● Proper labelling and upkeep of Equipments
● Careful supervision of students while doing practical work.
● Advance precautionary arrangements to meet any emergency situations.
● Conduct of any additional experimental work only under supervision and with due advance permission.
● Availability of First Aid and basic medical facilities in the school.
● Proper location of the laboratories.

How to make the learning of the difficult topics easier?


Do's Und Dont's
1. Do not take the word difficult while teaching and you be positive yourself.
2. Do not place the topic on the board - After completing tell the students that this “what
it is”— For example topics like Potentiometer
3. Try to build the topic from the basics of the basic while teaching.
4. Do not draw the diagram on board before you start the topic – Do build the same as
the discussion continues.
5. Do not forget to place the arrow in circuits and Ray diagrams – A mistake which can
be easily absorbed by the student.
6. Do not postpone the topic for the end of the academic session
7. Give 2-3 revision by asking question's from such identified topics at the beginning of
the class on the subsequent days.
8. Try to test these identified topics in almost all the tests if possible after prior information
to the students.
9. Do try to create interest on such topics before it is actually discussed.
10. Try to adopt an interactive approach to deal with such topics.
11. Very important a point is to think new and good approaches that may fit your students
while dealing with such topics and popularize such method.

(v)
Introducing Life Skills and Values in
Teaching of Physics

Aim of education is not merely to transfer the information but it is to improve the quality of life. Education
is inherently values oriented and must develop in learners caring, cooperation and respect for others. In
addition to equipping them with life skills and attitudes, it must prepare tham to lead a full life. As for as
learning of science in general and Physics specifically corner the teacher must motivate the students to
acquire and exercise values during the process of learning.
Science is often equated with facts, knowledge and experiments. The subject matter of science
examples one to develop critical thinking skills, decision making and problem solving skills.
While teaching science, developing scientific attitude is the main aim but a variety of human values
can also be developed. While speaking about the quality of a good instrument, qualities like sensitivity,
stability and consistency can be pointed out.
Further it promotes curiosity to learn more and develop scientific attitudes and emphasize how human
kind needs to stop exploiting nature to his advantage.
Students should be motivated to explore values based on life skills like self awareness, effective
communication, decision making ability, creative thinking, critical thinking, problem solving, empathy,
co-opration, managing emotions and coping with stress.
The values can be acquired every moment and every day of our lives but some basic parameters and
focus area are :
● During the process of socialization e.g. immediate family, friends, school teachers, peer group,
neighbourhood, etc.
● Contact with the broader community e.g. family network, mass media, work places, etc.
● Habits imbibed as we grow up e.g. prohibitions, commonds and identification with family and
friends, approval and disapproval of actions, deeds, etc.
● Obligations and norms of the environment around us e.g. shift from ‘must consciousness to most
consciousness’ I must not give way to anger reflect compulsion where as statements like I
might to respect may elders reflect the sens of obligation rather than fear or compulsion.
Values should not be treated as ideal concepts but as ‘empowering tools’ which are helpful in meeting
the challenges of the contemporary social world be it religious fundamentalism, environmental degradation
multicultural constricts, misuse of science and technology, inequalities, ill effects of mass media,
globalization, commercialization and so on. India is multilingual, multicultural and multi religious country.
The exploding world of science and technology has many good and bad unintended consequences.
How science and technology is to be used is a question of values. The question of social, moral responsibility
and the universal, ethical values occupy the natural position in application of science and technology in a
more human and rational way. The values and attitudes can only make difference to guide individual

(vi)
actions based on the principles of fairness, goodness, constructive and critical attitude and wisdom to
discern between the potential and pitfalls of scientific and technological advancements.
For instance in the science (Physics) that books there is topic on, refraction of light, and students
observe an illusion that a straight glass rod appears bent when placed in a glass number filled with water.
The understanding of processes of refraction and bending of lights convects that sometimes what we
are seeing with our own eyes many not be true, just as the glass rod half dipped in water in a glass tumbler
appears to be bent but it is only an illusion. As we learn move about properties of light, we know why rod
appears bent. Use examples of this kind to illustrate the phenomenon of cultural biases towards others.
Teachers can motivate the students by an interesting way during the teaching of core subjects for
example Menton’s first law or law of inertia states that an law or law of inertia states that an effort is
required to change the state whether it is the state of rest or state motion. Similarly if a change is required
in any state whether it is social, geographical, cultural, political or any desirable state it needs effort hence,
value of dignitary of labour can the introduced at this stage.
The task of a teacher is a class room is very beautifully described by Donald D. Quinn. He says, “If
a doctor, lawyer, or dentist had 40 people in his office at one time, all of whom had different needs and
some of whom did’t want to be there and were causing trouble, and the doctor, lawyer, or dentist, without
assistance, had to treat them all with professional excellence for nine months, then he might have some
conception of the classroom teacher’s job”.
A teacher therefore should be adept in reconciling the infinite variables into a viable class room
solution. It is important to know your learner, everyone of them. This pedagogically speaking a learner
would require understanding how each child constructs knowledge and forms concepts. Every time a learner
internalize some knowledge it an outcome of a process involving repeated observation, correlating to
construct an emerging pattern, drawing inference, adopting and applying, questioning for further discovers
and expansion. The algorithm of the process of learnign may be presented thus.”
● Each child at any stage starts with some knowledge.
● Allomodates new experiences by reconcilling / assimilating with the existing and reference.
Thus accommodations is the process of refraining one’s mental representation of external world
to fit new experiences.
● The child is not isolated but works in an interactive environment.
● The instructor is not absent, his role is to facilitate, create environment, to assess to orient if the
learner wanders or goes beyond.
It is often associated with pedagogy approaches that promote learning by doing. Hence along with
the cognitive development the effective domain must on to friend the learner for life skills and being out
the values.
Acknowledgement
● Central Board of Secondary Education
● Values Education
(A handbook teachers)
● NCF - 2005 (NCERT)

(vii)
Tips to Prepare Physics students for CBSE
Board Examination

The class 12 board exams are very important in the academic lives of all students. If the students get good
marks in this exam, the possibility for them to secure admission into the course of their choice will become
high. If the marks they secure are low, they will end up blemishing their entire career. Getting above 90
percent on the CBSE class 12 exams in PCM, in a nutshell, will be both rewarding as well as mind boggling.
Scoring high marks in CBSE board exams needs thorough preparation. You can prepare well for it with
thorough revisions and repeated practice of tackling questions asked from any part of the syllabus. The
marks are deducted for missing steps while writing answers in the Physics paper as per marking scheme
issued .Students must be shown the marking scheme and must be demonstrated ideal answer sheets which
can score hundred percent marks.
For the date sheet students should be advised to be connected and log on to the CBSE Website at
www.cbse.nic.in. The Sample Question Papers in all the subjects are also available on this site. However,
if the students want to purchase the Sample papers and Marking Schemes of Board’s exams the same are
available at the CBSE Headquarter, Preet Vihar, Delhi-110092 between 9.30AM to 1.00PM on any working
day.
Some of the important topics in Physics, for class 12 CBSE examinations along with marks
distributions are:
Units Marks Units Marks
1. Electrostatics 8 6. Optics 14
2. Current Electricity 7 7. Dual Nature of Radiation and Matter 4
3. Magnetic Effect of Current 8 8. Atoms and Nuclei 6
and Magnetism
4. Electromagnetic Induction 8 9. Electronic Devices 7
and Alternating Current
5. EM waves 3 10. Communication Systems 5

Optics – The unit is for 14 marks in the examination, Electrostatics, Magnetic Effect of Current and
Magnetic effects of current for 8 marks each, Electromagnetic Induction and alternating current and current
electricity for 8 marks and 7 marks respectively, Electronic Devices is for 7 marks while atoms and nuclei
is for 6 marks. We can see various types of questions in the Physics Question Paper of CBSE Examinations.
The types of questions expected from each units as per the Physics sample papers is demonstrated below
after analysis along with their weight age in percentage . This Question papers analysis is based upon
CBSE Board Examination pattern:

(viii)
Question Type Marks (%) Question Type Marks (%) Question Type Marks (%)

Definition Based 36 Derivation 13 Numerical 26


Formula Based 6 Law/Principle 6 Memory 13
(application)

Total Allotted Marks for Physics Paper is 70, therefore as per the above analysis student must study
definitions , solve derivations , memorize formulae, practice numerical, understand application of Laws
/ Principles as per the weight age given per unit above and prepare the topics in the syllabus accordingly.
Teachers must share with their students some of the Important Point which they must always follow while
preparing for physics board examination as given below :
1. Make a chapter-wise list of all the concepts, important SI units, formulae, constants, derivations and
the laws they have learned throughout the year.
2. Always go for mathematical expressions and technical terms while answering the questions.
3. Wherever possible give labeled diagrams to reduce the number of words in their answers.
4. Whenever draw a circuit, be careful with the direction of current and polarity of cells. In case of ray
diagrams ask them not to forget to give the direction of the rays and the label correctly.
5. Follow standard books for derivations. Do not try to invent new ways for any derivation in the
examination hall.
6. Always give step-wise solution to numerical because marks are allotted to each of the steps.
7. Always be specific with the answers. Neither write too much and nor too less.
8. Make your preparation syllabus-oriented. For last moment preparations, do not think about learning
something out of syllabus or new concept.
9. Since five-mark questions have different parts, therefore prepare as well as write the answers of such
questions part wise. Revise the applications of each concerned concept accordingly .
10. Bone up on the following topics as they carry almost 70% of the marks electrostatics, current electricity,
and magnetic effect of electric current, EMI & AC, optics, and semiconductor.
11. Prepare students to complete the paper on time by solving previous years question papers. Assess
their preparedness using the marking scheme.
12. Written practice is needed to be thorough with concepts.
13. Revise short and very short answer type questions from each unit.
14. Circuit diagrams, ray diagrams, graphs, and diagrams of devices must be practiced with proper
labelling and attention to minute details.
15. Ensure all the formulae are on students fingertips.
16. It would be very helpful to them if they make an all inclusive list of formulae from all the units.
17. The students can try out all the numerical type questions from each unit.

(ix)
18. Sample question papers will be available from the formal website of the CBSE. Students must be
given opportunity to try out these questions.
19. Make sure that students read every chapter thoroughly. The questions may be direct or indirect.
Knowing all the concepts will help the students answer effortlessly. Refer to Textbook Solutions for
complete solution to NCERT textbooks. It is the most important book for Physics board exam of
class 12. All topics in the syllabus for the exam are covered in the NCERT book.
20. Ask students to make short notes about derivations in each and every chapter. In addition to this,
writing down the proofs will also be good. Get complete preparatory materials from Exam Resources.
21. While solving a derivation, let the students try and comprehend the logic behind the derivations
22. If students do not like the numerical part, than ask them to get used to the numerical part through
continuous practice . Solve each and every numerical in the NCERT book (both, solved and unsolved)
and, also refer a some supplementary book. A Physics paper without a numerical is like a comb
without teeth.
23. Do not forget to mention the S.I units (if any) of all physical entities.
24. During the exam, ask students to try to write descriptive answers in points and give pictorial or
graphical illustrations wherever possible. It enhances visibility also .
25. Ask students to just read the whole question paper after receipt in examination hall before answering.
26. Ask students to study in a “slow and steady” manner.
27. Ask students to study in a calm and quiet environment.
28. Discuss students for long answer type questions, before starting, frame a skeleton of the answer in
the margin.
29. Ask students to be crisp and to the point in very short questions.
30. A student can easily score good marks if he/she sticks to NCERT books.
31. Evaluate their performance by appearing for mock tests (at least one exam daily).
32. Advice students to stop watching TV. An hour of TV watching will not refresh but will tire their
eyes & nerves.
33. Identify the chapters in which the weight age is more and Advice students to revise them again and
again
34. Improve their English as they may not be able to comprehend a question of a subject due to weak
English.
35. Advice students to be a hermit in study and avoid social engagements like marriages & birthdays up
to the examinations.
36. Advice students to focus towards the exam & exam only.
37. Advice students to learn answers by writing.
38. Advice students to not be disheartened anytime. A serious and honest effort will always bear good
fruits.

(x)
39. Advice students to avoid use of computer unless it is very necessary.
40. Advice students to always feel positive about themself.
41. Advice students to not to leave and once thoroughly prepare the low weight age topics like
Electromagnetic Waves, principles of communication etc .
42. Help them to understand to express all the definitions, interpretation of figures, advantages and
disadvantages of various devices and applications etc.

TIPS FOR STUDENTS TO OVERCOME EXAMINATION ANXIETY


01. Writing practice should be done by solving different questions.
02. Relaxation-practice deep breathing, yoga or any other relaxation technique may be tried with students
to improve concentration.
03. Quickest and most effective way of eliminating stress is to shut down eyes and take deep breaths.
04. As far as possible let them continue with the normal routine of sleeping and eating.
05. Take help of the councilor and parents from time to time.
06. Help and guide students to never fear from examinations, avoid panic, and most important, do not
worry about results. Give their best shots and move forward. Guide them to find a family member
or trusted friend with whom they can talk about the things that stress out about the exam. When this
person tells them that everything is going to be OK, than advice them to believe it. You can also play
that role very well .

TIPS TO STUDENTS TO LEARN MORE BY TAKING BREAKS


(a) Take short breaks while studying.
(b) Do not study continuously for more than 30 to 45 minutes.
(c) Give a break of five to ten minutes in between two sessions.
(d) Play some games that requires physical movements
(e) Take at least 6 to 7 hours of sleep to refresh.
(f) Do not study at a long stretch.
(g) Such short breaks give rest to students brain and help it to reinforce whatever they are learning. This
will make it easy for students to understand more and remember better.
(h) Do not read any new information during these short breaks. Just relax or walk around.

TIPS TO STUDENTS TO SCORE WELL IN EXAMINATIONS


Regarding scoring of good marks in Board Exams, it is not so difficult to prepare students to score well.
Students have to stick to following important tips:
1. Firstly, ask students to study regularly, take out time for at least three subjects in a day. Work more on
those subjects, in which they feel are weaker than other subjects.
2. Do not ask them to leave any subject for more than 2-3 days. Every subject should be taken up for
study at least in every alternate day. The important and difficult ones may be taken up every day.

(xi)
3. Students must devote at least 6-7 hours for self study. But do not ask them to cut down too much on
sleep and enjoyment. Let them take out a bit of time for relaxing and entertaining also.
4. Whenever study, don’t ask them to sit for too long continuously, they can also take one or two short
breaks of 5-10 minutes during a continuous stretch of 3-4 hrs. of study as already mentioned above.
This will help in maintaining interest.
5. Whenever you give a test in school and give marks, just ask them to go through answer sheet
seriously and see where marks have been deducted and how can the errors be avoided next time..
6. At the end of the year, ask them to do at least 8-10 model test papers for each subject. This will help
a lot.Just keep working towards goal and keep on evaluating themself.
7. Never leave any question.Write something of what you know of the answer.
8. When you approach the numerical questions, always understand the question recall the known concept
of the question and always mention the formulae and unit.
9. Before you begin to write in exams,spend a few minutes to brainstorming the ideas and outlining the
arguments you want to make.

TIPS FOR STUDENTS TO INCREASE CONCENTRATION


1. Start assignments with some curiosity about the material and a positive attitude toward learning.
2. Designate a place where you go only to study. Use proper lighting.
3. Identify your distractions. Find ways to decrease them or to postpone them until study breaks (e.g.,
taking the phone off the hook, turning off instant messenger etc.).
4. Decrease noises around you while studying. If you need some background music it should be soft.
Keep the TV radio,mobile, mp-3 player etc in off position.
5. Use “active study” techniques: sit straight in a chair at a desk, start out with questions about the
material, outline chapters, underline key phrases after reading a section, write notes in margins, ask
yourself whatever you have learned.
6. Divide your work into smaller manageable tasks that can be completed in a short period of time.
Push yourself to complete one small task, then move on to the next task. Focus on one small task at
a time.
7. Use times of peak alertness for studying difficult or less interesting topics. When you are tired or
hungry, concentration will be lowered.
8. When your mind starts to wander come up with some cue words to say to yourself (e.g., “Focus.”
“Get back on task.”) to focus your concentration again.
9. If you have other assignments or issues on your mind write them down on a “to do” list or take a
small step to deal with them. Then get back to focusing on the task at hand.

TIPS TO STUDENTS TO FOLLOW STUDY TIME TABLE


Students must make a realistic time table for study i.e. the time table should be one, which can be followed.
If you have only 6 hours spare for study and you want to devote all those hours for self study or you have
made a time table which can not be followed for a longer time, avoid that.

(xii)
1. Make a flexible time table for study with some rigidity. For example, if you want to devote 2 hours
daily for a particular subject, say science, then try to maintain the time of 2 hours everyday without
fixing a rigid time.
2. Try to stick to schedules. For example, If you have decided to devote a total of 5 hours for self study,
try to stick to that and study at least that much.
3. The time table should be such that it is open to changes with time. After following the time table for
few days, if you feel, overall time period or time period for a particular subject are improper then
change them according to your requirements.
4. Initially, don’t make exhaustive time table for self study. Increase your study timings every week.
For example, if you are studying 3 hours in a week, try to increase 15 minutes in first week, 15-20
minutes in next week and so on
5. Allotting time for different subjects is the best and essential for studies.
6. During framing time table one has to keep in mind the availability of the time and other factors.
7. Sleep early the night before, get up early the next morning.
8. The best time of study is the morning time.

IMPORTANCE OF GOOD HANDWRITING IN EXAMS


Good handwriting always gives an upper hand to a student, but that doesn’t mean that poor handwriting
always costs. Actually what matters is writing neatly, make sure that examiner understands what student
have written. Some of the important tips in this direction are :
1. Ask students to use Black pen to write title and subtitles.
2. Advice students to make diagrams wherever necessary.
3. Wherever possible students must use scale for drawing.
4. Students should underline the key words.
5. Ask students to always start a new question on a new page.
6. If there is any mistake students should not make it look shabby by scratching it many times rather
just put a line over it.
Ask students to follow the above points which will definitely earn 5 to 6 marks more than expected.

BALANCED DIET AND PHYSICAL HEALTH


The bulk of evidence supporting the effectiveness of these so-called brain foods in improving one’s study
skills consists of testimonials or anecdotal evidence; but we’re going to enumerate several nutrients that
can help students to improve their study skills –
1. Have a balanced diet. drink lot of water, avoid oily foods and have following minerals:
(a). Some studies have shown that there is a connection between memory loss and lack of vitamin
E.It is common for vitamin E to be included in the supplement staples of senior citizens and
strock victims.

(xiii)
( b). Peanuts are also considered as brain food. Peanuts are high in protein and proteins are known
as the building blocks of our organs and muscles – brain muscles.
(c). Coffee and colas with high caffeine help because the caffeine keeps you awake; other than
that, there is not much basis to consider them brain foods. But don’t drink so much because it
may blur the capacity to think clearly.
2. Avoid getting ill and take all the necessary precautions to remain fit and healthy.
3. Do relaxation practice, deep breathing, yoga, meditation and exercises because it helps in improving
concentration and it boosts the energy.
4. The diet should include nutrients rich food items like whole grains, nuts, fruits vegetables and milk.

HOW TO USE NEW KNOWLEDGE


Whenever students are studying any new concept or unit, Ask students to stop to imagine how they will use
whatever they have learnt. There is always so much information, and yet little of it is the “important stuff.”
But by imagining how they will use new information, they tend to automatically focus on the things really
need to know.

DIFFICULTIES THAT STUDENT EXPERIENCES


Examinations appear to be terrifying. Appearing in the exam strikes terror into the minds, but there is no
way out. But for some students exams are enjoyable experience. These are those categories of students who
are consistent and up to date in their studies. They are self-confident and well prepared for the examinations.
Therefore one should not fear from it. The only way is to prepare for it. But there are many situations or
problems that a student faces during preparing for examinations.
Some of the reasons are:- Lengthy courses- Nowadays, courses are very lengthy. On one subject
there are many books that are required to study. A science stream student is required to study physics,
chemistry, mathematics and sometimes computer science, biology. Student has to go to the laboratory for
doing experiments regarding their subjects. Thus he fails to cope with other subjects. For this it is necessary
to maintain notes and consistency from staring.
Students start preparation late- Its better to start studies from the beginning of the session. By doing
regular studies, the fear of the exams vanishes. Students who start preparation late get confused with which
subject to start with and gets nervous.

IMPORTANT TIPS TO STUDENTS TO STUDY FOR EXAMINATION


Scoring in Examination is a pure art. Students those try hard to study every material of a topic for examination,
sometimes get confused and don’t get marks for the examination. Those who study little, but systematically
with important questions get more and better marks. The difference lies in the method of preparation for
the examination. Intelligent preparation and memorization, and not a thoughtless cramming results in
success. There are some tips for the students to score well in examinations. In spite of good preparation and
practice, students not able to perform well or give their 100%. There are some points to follow and remember
before appearing in the CBSE Board examination.

(xiv)
1. Students don’t start paper just after receiving it, first read it carefully, give little time in understanding
and deciding for where and what to start.
2. Some students attempts paper serial wise, its good but if you don’t know a particular question don’t
attempt it and mark that one with it as un attempted. It is advised to attempt only those in which you
are more confident. This will not only save your time but also make you confident and minimize
your stress as well.
3. There would be some students who are still not feeling confident; those students are advised to try to
find out the main root of your problem before the examination. Why are you not confident and try to
solve that?
4. In case of too much stress the best and simple solution is to shut down your eyes and take deep
breaths.
5. Students should not waste time in solving repeated questions and use every second of examination
time effectively.
6. Those who are already tense and in taking too much stress should not cut down their sleep. As
healthy Body = healthy mind
7. students should never fear with exams rather, board exams are not different; these are just like
school exams. Just do your best and don’t worry for results. Prepare for the next without worrying
for the past one.
8. All the student must be asked to check carefully not to forget their examination material at home
(eg. Roll Number, I Card , pen, pencil, eraser, geometry box, etc .Each students must be advised
before leaving for the exam check for all of the important things .
9. Proper planning is required for the board examination to score 100%. So each students must plan it
before the examination

STUDY TIPS FOR STUDENTS


● Guide students to enrich their environment with light, inspiring messages, flowers and plants etc .
● There is no substitute for self-control and discipline. It is not a restriction of freedom. Instead, it
enables students to be a confident learner.
● Ask students to be persistent.
● Always advice students to analyse their task themselves and divide it into smaller study tasks to
experience success.
● Ask them and help to use their intelligence to create study tools to learn and memorise.

TIPS ON HOW TO REVISE


● Keep aside enough time to revise so that students don’t get into a situation of last minute cramming.
This approach will boost your confidence and reduce any pre-exam stress
● Experiment with alternative revision techniques so that revising is more fun and self motivation to
study is high

(xv)
● “We learn more by looking for the answer to a question and not finding it than we do from learning
the answer itself.” – Lord Alexander
● Exams, be it at the entrance level, university level or even at the school level, can be big causes of
stress. The key to performing well in the exams, apart from, studying hard, grasping everything and
retaining everything, is, being able to study smart and in a proper manner, so that students efforts
translate into performance.
● Don’t have too much on your plate: Doing exercises to use brain in a constructive way, and do not
clutter it with a lot of thoughts is very important . Student must be clear in his mind and do not
pressure himself to cover things that are beyond his reach. Make realistic targets for a day. Even
Rome was not built in a day.Do not overdo things. Do not try and study everything in a single day. Of
the 5 or 6 subjects, pick up 2 or a maximum of 3 subjects and pick up a topic or two from each and
go into the depths of the topics, instead of skimming 8-10 topics.
● Solve papers: It is often said that there is no better practice than match practice. Solve one paper
each day, preferably from 10 AM to 1 PM, in an environment as close to the one you expect at your
examination centre. After you have solved the paper, mark yourself as per the CBSE marking scheme.
Pick a different subject for each of the five/six days. Assess yourself at the end of the week, look at
your weak areas and try to rectify them in the weeks to follow. While solving sample papers, make a
point to solve the ones provided by the CBSE. The papers provided by the CBSE are the closest
possible papers to the real thing available to all students .
● Exercise and meditate: Exercise and meditation help improve concentration. Many of the athletes
and sports professionals regularly employ meditation methods. Studies have found a direct correlation
between concentration exercises (meditation) and the performance level of sports professionals.
Meditation strengthens the mind, it comes under control and is able to provide effective guidance to
the physical body to effectively execute all its projects. Psychological Exercises are a powerful way
of improving concentration and improving mental strength.

HOW TO PREPARE IF YOU HAVE NOT DONE WELL IN YOUR PRE-BOARDS


● Remember, pre-boards are an indicator, not the real thing. Students still have time to better themself
for the final frontier.
● Such students must devote more time to weak areas and solve questions of varying difficulty. They
should not solve all easy questions or all tough questions at a time . Easy questions will motivate
them ,but when they are unable to solve difficult questions might lead to dishearten or loose of
confidence and not best possible solution. All such students are advised to gradually increase the
level of difficulty of the questions they are attempting, analogous to climbing a staircase.
● Take sectional tests for the areas students are not comfortable with. Once they start to feel confident
with weak areas, start taking full length tests in a timed environment.
● The first thing the student must do is to stop being worried. Becoming agitated will only increase the
stress. This won’t provide the students with the right mind to study. The students must give more
importance to the physics subject. The basic thing they need to understand the subject more is to be
more aware of its concepts.

(xvi)
● The best method to gain this knowledge is to keep on reading the physics text at all times. It would
also do them some good if they write down shorts notes when they are studying. This will be of help
for them prior to their finals. They won’t have to read the entire notes, but can go through these short
notes and understand the concepts really quickly.

HOW TO PREPARE IF STUDENTS HAVE DONE WELL IN PRE-BOARDS


● If the exam was a success, the students must never take this to their heads. The model test is only a
prelude to the finals. The latter will be tougher than the former. Students can prepare themselves by
studying from the top. Do not lose focus. Remember that the real test is not over, yet.
● They should try learning each portion as many times as possible. Apart from this, they can also find
question papers online. Interactive quizzes as well as well revised notes will be available for the
students to use. These effective online revisions will help the students improve their time management
skills
● Do not allow such students to be overconfident. Now that they have done well in pre-board exams,
do not let them to be complacent and lethargic. It’s the big responsibility to guide such students to
treat their pre-board result like something that motivate them to score more, maintain record and
performance and keep improving their results.

CONCLUSION
So, when you look at the whole preparation , the final month is important for everyone. The ones who have
their preparation in fourth gear can still improve it by putting it in overdrive and the ones who have some
doubts can still improve by concentrating on their weak areas while simultaneously focusing on their areas
of strength.
During this time, the aim of a student should be, to perform, to the best of his/her abilities and also
concentrating on keeping himself healthy. Like Lord Krishna said, “Karmanye Vadhikaraste Ma Phaleshu
Kadachana,”, you should concentrate on performing to the best of your abilities while not worrying much
about the end results

LAST-MINUTE TIP TO SHARE WITH STUDENTS FOR BOARD EXAMINATIONS


Believe in yourself: If you have prepared well for the exam, do not worry unnecessarily. Only, make sure
you have the right perspective. If you think that anything less than 90% is a failure, you are creating
unnecessary stress for yourself. Don’t bottle up: Confiding in someone you trust is a great way of alleviating
stress. Right perspective: The board exam may seem to be the most important thing right now, but in the
bigger scheme of your life its plays a small role.

Students can be guided and motivated through the following lines ..


“Challenges are high, dreams are new,
The world out there is waiting for you,
Dare to dream, Dare to try,
No goal is too distant, No star too high.”

(xvii)
Activities to Create Interest in the Subject

Science is not in the books.


Science is not in the apparatus
Science is not in the activities
Science is in doing experiments/activities with a view to realizing laws and concept of science.

Science is everywhere when Science is in attitude


The only natural, least time consuming and joyful method to understand science is learning by doing.
Though all the activities included in the syllabus are well defined like experiments as regard to the objectives/
aims to be achieved, yet we have freedom to go to any extent in respect of time, the materials to be used and
procedure to be followed. Therefore activities provide ample scope for Innovations.

CLASS XII

Activity 1
Aim: To measure the resistance and impedance of an inductor with and without iron core
Concepts involved
I. An Inductor (a copper/ aluminum coil or solenoid) offers very low ohmic resistance in a steady D.C.
circuit.
II. In an A.C. or varying D.C. circuit an inductor offers both ohmic and non-ohmic resistance. Non-
ohmic resistance called reactance (XL = 6.28 f L) depends upon the frequency of a. c., geometry of
the coil and nature of the core used.
III. There is phase difference of 90° between current and voltage across inductor. Therefore R and XL are
perpendicular to each other.

Material required
A.C. (6-12 v) source or variac, inductor having more than 2000 turns of copper wire (34 or 36 SWG), AC
ammeter, DC Ammeters and Voltmeters.
(Soft iron core can be made by stacking ‘I’ shaped cores of a burnt out step down transformer)
(If the activity is performed using varying D.C., then we require only D.C. meters for the current and
the voltage measurements)

Circuit set up

(xviii)
Measurement of resistance
S.N. Ammeter reading I voltmeter reading V R=

Z = R 2 + (ωL) 2 without iron core; Z’ = R 2 + (µ ωL) 2 with iron core


Conclusion: Impedance of inductor with iron core is much higher than without it

Activity-2
Aim: To assemble a household circuit comprising three bulbs, three on / off switches a fuse and a power
source.

Concept Involved
I. Household electric circuit is a constant voltage circuit; all the gadgets/ appliances are connected in
parallel.
II. Current through a gadget (fan, bulbs etc.) Is inversely proportional to its impedance.
1
I ∝
z
IIl. Fuse (overload protection) is connected in series in live wire.
IV. Provision of earth wire for appliances having metallic body
For a better understanding of a household electric Circuit, two separate circuits for 5A and 15A should be
assembled on ply boards.

While assembling circuit use:


1.5 mm wire for 5A circuit
2.5 mm wire for 15A circuit
0.7 mm wire (Green) for earth connection
4.0 mm wire for A.C. Socket for air conditioner
Diagram for 5A circuit

5A 5A

(xix)
Activity — 3
Aim : To study the variation in potential drop with length for a steady current
Concept used:
Potential drop across a resistance wire due to steady current varies—
— directly as length of the wire and its resistivity
— inversely as its area of cross section
For a wire of homogeneous material (uniform resistivity) and uniform thickness, potential drop is proportional
to the length of the wire
Circuit Diagram

A Galvanometer with a series resistance (1000 Ω) can be used in place of a voltmeter Resistance
wire AB about 1m in length is fixed on a ply board using nails.
Activity should be repeated by taking two are more resistance wires of different materials and different
thickness.

Activity — 4
Aim: - To Study the effect of intensity of light (by varying distance of the source) on an L. D. R.

Concept used
The resistance of certain semiconductors such as cadmium sulphide decreases as the intensity of light
falling on them increases.
Light sets free electrons from donor atoms in the semiconductor, so increasing its conductivity.

Circuit Diagram

(xx)
Resistance of LDR varies from 10 m Ohm in the dark to 1 k Ohm or so in daylight. LDR is used in light
operated on/off switch.
(Circuit diagram of a light operated switch is shown below)

Activity — 5
Aim : To observe polarization of light using Polaroids

Concept used
Only transverse waves exhibit phenomenon of polarization.
Since light is electromagnetic transverse wave, therefore it can be polarized.

Material
This activity can be done by using easily available low cost pencil torch as a source of light.
Two sheets of Polaroid can be obtained from a damaged or useless digital calculator or a digital watch.

Procedure
Using the torch make a bright spot on a white screen.
Insert Polaroid/ Polaroids in the path of the beam to observe polarization of light.
Polarization of light by reflection can also be observed.

Activity — 6
Aim : To identify a diode, an LED, a resistor and a capacitor
To test continuity of a cartridge fuse
To identify base of transistor
To distinguish between NPN and PNP type transistors
All the above activities can be easily performed by using 2 cells of 1.5v each, 330 Ohm 1/2watt
resistor and 3V LED.
Current limiting
Concept used resistor

An LED consists of a junction diode made from the semiconducting


compound gallium arsenide phosphide. LED emits light when
forward biased.

(xxi)
Procedure
Lead X is positive and lead Y is negative.
(1) When P end of a semiconductor diode or a transistor is connected to X terminal and N end is connected
to Y terminal LED emits light. When the leads of the diode or the transistor are interchanged, LED
does not glow.
(2) When a conductor (resistor, cartridge fuse etc.) is connected between X & Y the glow of the LED is
reduced according to the value of resistance offered by the conductor.
(3) When a capacitor is connected between, X & Y the LED glows only for a short duration depending
on the time constant.
LED will not emit light for a very low value capacitor.

CLASS XI

Activity
Aim : To observe and explain the effect of heating on a bimetallic strip

Concept involved
Metals expand on heating due to increase in the amplitude of oscillations of molecules and atoms.
Different materials show different expansion. Bimetallic strip gets curved when heated. The metal,
with larger value of coefficient of linear expansion in the strip, remains on the outside of the curved strip.

Material
A small size bimetallic strip can be obtained by gently breaking the neon bulb used in the fluorescent tube
starter or master bulb used in blinking low voltage series bulb arrangement.
Shape of the bimetallic strip obtained is as shown under

ABC is Bimetallic Strip. When heated, strip ABC bends and makes contact with EF.
Applications: A bimetallic strip can be used to make a fire alarm, heat operated on/off switch, a
thermostat etc.

(xxii)
Make the bent bimetallic strip straight and attach a long pointer. Now the bimetallic strip can function
as thermometer.

Bimetallic strip in the shape given below can work as THERMOSTAT. (Heat to off circuit)

(xxiii)
Useful Information about Problem Solving
Assessment (PSA) for Class-XI

CBSE PSA Class-XI


1. For assessing the problem solving skills of the students and to prepare them to face the twenty first
century, Problem Solving Assessment (PSA) has been introduced by CBSE vide circular no. Acad-
41/2012 dated August 1, 2012.
2. It will be compulsory for all students of class XI and carry 90 marks. There will be 60 items of MCQ
type. There in no specific syllabus for PSA.
3. It will assess the following areas :
a. Quantitative Reasoning (found in Mathematics , Science and Technology)
b. Qualitative Reasoning (found in humanities , arts and social sciences)
c. Language Conventions
4. PSA will help the students in developing skills such as problem solving, decision making, critical
and creative thinking. This in turn, will help in developing Higher Order Thinking Skills (HOTS).It
will help in developing these skills even in an average student. Children should be able to assess
student’s ability to process , interpret and use information rather that student’s prior subject matter
knowledge.
5. The Assessment will contain items that will assess written expression, including grammar and usage,
vocabulary in context and passage-completion.
6. Separate registration for appearing for PSA will not be required for the students.
7. Question paper for PSA will be sent to the schools one week before the examination by the Board.
8. The question paper booklets carrying the code 111P are for Class 11 students.

Pattern of examination:
● One question paper containing qualitative and quantitative reasoning and language conventions.
❍ Ninety marks in total are distributed as follows:
Qualitative reasoning – Stand alone MCQs, Passage based MCQs (Sets 1 and 2) (6 items in each)
Quantitative reasoning – Stand alone MCQs (10 items) and Passage based MCQs (Sets 1 and
2 with 4 items in each)
Language conventions – Grammar and usage MCQs, Vocabulary in context based MCQs and
Passage completion based MCQs (8 items in each)
OMR sheet will be attached to the question booklet.
● Irrespective of the subject chosen by the student, each one of them will be given the same question
paper.

(xxiv)
7. The dyslexic, spastic and autistic candidates as well as those with visual and hearing impairment have
the option of offering one language and any four subjects at secondary level from Mathematics,
Science, Social Science, another language, music, home science, foundation of information technology,
Commerce (Elements of Business) and Commerce (Elements of Bookkeeping and Accountancy).
Such students should be provided relaxation from attempting those questions from subjects they have
not opted for at the secondary level. For such students, the questions from all other subjects will be
counted as a total of 90 and scores will be given on the basis of that.
8. The examination will be conducted in the school.
9. Based on the scores obtained in PSA, the student will be given a certificate.
10. There will be no separate time tables or periods for teaching or practice of PSA in schools.

11. The quantitative domain of the CBSE – PSA would make comparatively modest domains on
mathematical knowledge but would emphasize logical and numerical reasoning and use of basic
mathematical knowledge.

References :
www.cbse.nic.in
http://www.cbseacademic.in/web_material/Notifications/2012/28_PSA-Additional_info.pdf
www.edudel.nic.in

(xxv)
Additional Information in relation to conduct of
Problem Solving Assessment for classes IX and XI

(a) No separate registration for PSA is required. Students already registered for class IX and XI with
the Board are eligible for this test and will use same registration number for PSA. There is no separate
roll number required for this purpose.

(b) The Board will make the question paper for PSA available to every school approximately one week
before the actual date of examination. The number of question papers to be sent to any school will
be in accordance with the total number of students registered for classes IX and XI.

(c) There will be a single question paper covering qualitative reasoning, quantitative reasoning and
language conventions as per the details given in the earlier circulars. The OMR answer sheet to be
used for answering the questions will be kept inside the question paper booklet. Sample question
paper for PSA will be made available on Board’s website by last week of November,2012. You may
download this material and make it available to all concerned.

(d) Similarly, every student of class-XI will get same question paper, irrespective of the subject combination
he/she is offering. However, this question paper will be different from the question paper used for
class-IX.

(e) As mentioned in the earlier circulars, the Problem Solving Assessment score will be counted towards
FA-4 of class-IX. This same score will be reflected in one language (English or Hindi), Mathematics,
Science and Social Science. Regular FA-4 will be conducted in the other language. Since there is
no FA-4 in class-XI , the students will be given a separate certificate in respect of their score obtained
in PSA. The choice of one language for PSA will be take jointly by students and school authorities.

(xxvi)
(xxvii)
(xxviii)
(xxix)
(xxx)
(xxxi)
(xxxii)
(xxxiii)
(xxxiv)
(xxxv)
PHYSICS (Code No. 042)

Senior Secondary stage of school education is a stage of transition from general education to discipline-
based focus on curriculum. The present updated syllabus keeps in view the rigour and depth of disciplinary
approach as well as the comprehension level of learners. Due care has also been taken that the syllabus
is comparable to the international standards. Salient features of the syllabus include:
l Emphasis on basic conceptual understanding of the content.
l Emphasis on use of SI units, symbols, nomenclature of physical quantities and formulations as
per international standards.
l Providing logical sequencing of units of the subject matter and proper placement of concepts
with their linkage for better learning.
l Reducing the curriculum load by eliminating overlapping of concepts/ content within the discipline
and other disciplines.
l Promotion of process-skills, problem-solving abilities and applications of Physics concepts. Besides,
the syllabus also attempts to
l strengthen the concepts developed at the secondary stage to provide firm foundation for further
learning in the subject.
l expose the learners to different processes used in Physics-related industrial and technological
applications.
l develop process-skills and experimental, observational, manipulative, decision making and
investigatory skills in the learners.
l promote problem solving abilities and creative thinking in learners.
l develop conceptual competence in the learners and make them realize and appreciate the interface
of Physics with other disciplines.

(xxxvi)
PHYSICS
COURSE STRUCTURE
2011-13
Class XI (Theory)
One Paper Three Hours
Max Marks: 70

Class XI Weightage

Unit I Physical World & Measurement 03


Unit II Kinematics 10
Unit III Laws of Motion 10
Unit IV Work, Energy & Power 06
Unit V Motion of System of particles & Rigid Body 06
Unit VI Gravitation 05
Unit VII Properties of Bulk Matter 10
Unit VIII Thermodynamics 05
Unit IX Behaviour of Perfect Gas & Kinetic Theory of gases 05
Unit X Oscillations & Waves 10
Total 70

Unit I: Physical World and Measurement (periods 10)


Physics - scope and excitement; nature of physical laws; Physics, technology and society. Need for
measurement: Units of measurement; systems of units; SI units, fundamental and derived units.
Length, mass and time measurements; accuracy and precision of measuring instruments; errors in
measurement; significant figures. Dimensions of physical quantities, dimensional analysis and its
applications.

Unit II: Kinematics (Periods 30)


Frame of reference, Motion in a straight line: Position-time graph, speed and velocity.
Elementary concepts of differentiation and intergration for describing motion.Uniform and nonuniform
motion, average speed and instantaneous velocity. Uniformly accelerated motion, velocitytime,
position-time graphs.
Relation for uniformly accelerated motion (graphical treatment).
Scalar and vector quantities; Position and displacement vertors, general vectors and notation; equality
of vectors, multiplication of vectors by a real number; addition and subtraction of vectors. Relative
velocity.

(xxxvii)
Unit vector; Resolution of a vector in a plane - rectangular components. Scalar and Vector product of
vectors. Motion in a plane. Cases of uniform velocity and uniform acceleration-projectile motion.
Uniform circular motion.

Unit III: Laws of Motion (Periods 16)


Intuitive Concept of force. Inertia, Newton’s first law of motion; momentum and Newton’s second
law of motion; impulse; Newton’s third law of motion. Law of conservation of linear momentum
and its applications.
Equilibrium of concurrent forces. Static and kinetic friction, laws of friction, rolling friction,
lubrication.
Dynamics of uniform circular motion: Centripetal force, examples of circular motion (vehicle on
level circular road, vehicle on banked road).

Unit IV: Work, Energy and Power (Periods 16)


Work done by a constant force and a variable force; kinetic energy, work-energy theorem, power.
Notion of potential energy, potential energy of a spring, conservative forces: conservation of
mechanical energy (kinetic and potential energies); non-conservative forces: motion in a vertical
circle; elastic and inelastic collisions in one and two dimensions.

Unit V: Motion of System of Particles and Rigid Body (Periods 18)


Centre of mass of a two-particle system, momentum conservation and centre of mass motion. Centre
of mass of a rigid body; centre of mass of uniform rod.
Moment of a force, torque, angular momentum, conservation of angular momentum with some
examples.
Equilibrium of rigid bodies, rigid body rotation and equations of rotational motion, comparison of
linear and rotational motions; moment of inertia, radius of gyration.
Values of moments of inertia, for simple geometrical objects (no derivation). Statement of parallel
and perpendicular axes theorems and their applications.

Unit VI: Gravitation (Periods 14)


Keplar’s laws of planetary motion. The universal law of gravitation.
Acceleration due to gravity and its variation with altitude and depth.
Gravitational potential energy; gravitational potential. Escape velocity. Orbital velocity of a satellite.
Geo-stationary satellites.

Unit VII: Properties of Bulk Matter (Periods 28)


Elastic behaviour, Stress-strain relationship, Hooke’s law, Young’s modulus, bulk modulus, shear,
modulus of rigidity, poisson’s ratio; elastic energy.
Pressure due to a fluid column; Pascal’s law and its applications (hydraulic lift and hydraulic brakes).
Effect of gravity on fluid pressure.
Viscosity, Stokes’ law, terminal velocity, Reynold’s number, streamline and turbulent flow. Critical
velocity. Bernoulli’s theorem and its applications.

(xxxviii)
Surface energy and surface tension, angle of contact, excess of pressure, application of surface tension
ideas to drops, bubbles and capillary rise.
Heat, temperature, thermal expansion; thermal expansion of solids, liquids and gases, anomalous
expansion; specific heat capacity; Cp, Cv - calorimetry; change of state - latent heat capacity.
Heat transfer-conduction, convection and radiation, Qualitative ideas of Blackbody radiation green
house effect, thermal conductivity, Newton’s law of cooling, Wein’s displacement Law, Stefan’s law.

Unit VIII: Thermodynamics (Periods 12)


Thermal equilibrium and definition of temperature (zeroth law of thermodynamics). Heat, work and
internal energy. First law of thermodynamics. Isothermal and adiabatic processes.
Second law of thermodynamics: reversible and irreversible processes. Heat engines and refrigerators.

Unit IX: Behaviour of Perfect Gas and Kinetic Theory (Periods 8)


Equation of state of a perfect gas, work done in compressing a gas.
Kinetic theory of gases - assumptions, concept of pressure. Kinetic energy and temperature; rms
speed of gas molecules; degrees of freedom, law of equipartition of energy (statement only) and
application to specific heat capacities of gases; concept of mean free path, Avogadro’s number.

Unit X: Oscillations and Waves (Periods 28)


Periodic motion - period, frequency, displacement as a function of time. Periodic functions. Simple
harmonic motion (S.H.M) and its equation; phase; oscillations of a spring–restoring force and force
constant; energy in S.H.M. Kinetic and potential energies; simple pendulum–derivation of expression
for its time period; free and forced and damped oscillations (qualitative ideas only), resonance.
Wave motion. Transverse and longitudinal waves, speed of wave motion. Displacement relation for
a progressive wave. Principle of superposition of waves, reflection of waves, standing waves in
strings and organ pipes, fundamental mode and harmonics, Beats, Doppler effect.

Practicals
Note: Every student will perform 15 experiments (8 from Section A and 7 from Section B).The
activities mentioned are for the purpose of demonstration by the teachers only. These are not to be
evaluated during the academic year. For evaluation in examination, students would be required to
perform two experiments - One from each Section.

(xxxix)
SECTION A
Experiments Total Periods : 60
(Any 8 experiments out of the following to be performed by the Students)
1. To measure diameter of a small spherical/cylindrical body using Vernier Callipers.
2. To measure internal diameter and depth of a given beaker/calorimeter using Vernier Callipers and
hence find its volume.
3. To measure diameter of a given wire using screw gauge.
4. To measure thickness of a given sheet using screw gauge.
5. To measure volume of an irregular lamina using screw gauge.
6. To determine radius of curvature of a given spherical surface by a spherometer.
7. To determine the mass of two different objects using a beam balance.
8. To find the weight of a given body using parallelogram law of vectors.
9. Using a simple pendulum, plot L-T and L-T2 graphs. Hence find the effective length of second’s
pendulum using appropriate graph.
10. To study the relationship betwen force of limiting friction and normal reaction and to find the co-
efficient of friction between a block and a horizontal surface.
11. To find the downward force, along an inclined plane, acting on a roller due to gravitational pull of the
earth and study its relationship with the angle of inclination (O) by plotting graph between force and
sinq.
Activities (For the purpose of demonstration only)
1. To make a paper scale of given least count, e.g. 0.2cm, 0.5 cm.
2. To determine mass of a given body using a metre scale by principle of moments.
3. To plot a graph for a given set of data, with proper choice of scales and error bars.
4. To measure the force of limiting friction for rolling of a roller on a horizontal plane.
5. To study the variation in range of a jet of water with angle of projection.
6. To study the conservation of energy of a ball rolling down on inclined plane (using a double inclined
plane).
7. To study dissipation of energy of a simple pendulum by plotting a graph between square of amplitude
and time.

(xxxx)
SECTION B
Experiments
(Any 7 experiments out of the following to be performed by the students)
1. To determine Young’s modulus of elasticity of the material of a given wire.
2. To find the force constant of a helical spring by plotting a graph between load and extension.
3. To study the variation in volume with pressure for a sample of air at constant temperature by
plotting graphs between P and V, and between P and I/V.
4. To determine the surface tension of water by capillary rise method.
5. To determine the coefficient of viscosity of a given viscous liquid by measuring terminal velocity
of a given spherical body.
6. To study the relationship between the temperature of a hot body and time by plotting a cooling
curve.
7. To determine specific heat capacity of a given (i) solid (ii) liquid, by method of mixtures.
8. (i) To study the relation between frequency and length of a given wire under constant tension
using sonometer.
(ii) To study the relation between the length of a given wire and tension for constant frequency
using sonometer.
9. To find the speed of sound in air at room temperature using a resonance tube by tworesonance
positions.

Activities (For the purpose of demonstration only)


1. To observe change of state and plot a cooling curve for molten wax.
2. To observe and explain the effect of heating on a bi-metallic strip.
3. To note the change in level of liquid in a container on heating and interpret the observations.
4. To study the effect of detergent on surface tension of water by observing capillary rise.
5. To study the factors affecting the rate of loss of heat of a liquid.
6. To study the effect of load on depression of a suitably clamped metre scale loaded at
(i) its end (ii) in the middle.

SUGGESTED LIST OF DEMONSTRATION EXPERIMENTS


CLASS XI
1. To demonstrate that a centripetal force is necessary for moving a body with a uniform speed along a
circle, and that the magnitude of this force increases with increase in angular speed.
2. To demonstrate inter-conversion of potential and kinetic energy.
3. To demonstrate conservation of linear momentum.
4. To demonstrate conservation of angular momentum.
5. To demonstrate the effect of angle of launch on range of a projectile.

(xxxxi)
6. To demonstrate that the moment of inertia of a rod changes with the change of position of a pair of
equal weights attached to the rod.
7. To study variation of volume of a gas with its pressure at constant temperature using a doctors’ syringe.
8. To demonstrate Bernoulli’s theorem with simple illustrations
9. To demonstrate that heat capacities of equal masses of different materials are different.
10. To demonstrate free oscillations of different vibrating systems.
11. To demonstrate resonance with a set of coupled pendulums.
12. To demonstrate longitudinal and transverse waves.
13. To demonstrate the phenomenon of beats, due to superposition, of waves produced by two sources of
sound of slightly different frequencies
14. To demonstrate resonance using an open pipe.
15. To demonstrate the direction of torque.
16. To demonstrate the law of moments.
Recommended Textbooks.
1. Physics Part-I, Textbook for Class XI, Published by NCERT
2. Physics Part-II, Textbook for Class XI, Published by NCERT

Class XII (Theory)


Total Periods : 180
One Paper Time: 3 Hours 70 Marks

Unit I Electrostatics 08
Unit II Current Electricity 07
Unit III Magnetic effect of current & Magnetism 08
Unit IV Electromagnetic Induction and Alternating current 08
Unit V Electromagnetic Waves 03
Unit VI Optics 14
Unit VII Dual Nature of Matter 04
Unit VIII Atoms and Nuclei 06
Unit IX Electronic Devices 07
Unit X Communication Systems 05
Total 70

Unit I: Electrostatics (Periods 25)


Electric Charges; Conservation of charge, Coulomb’s law-force between two point charges, forces
between multiple charges; superposition principle and continuous charge distribution.
Electric field, electric field due to a point charge, electric field lines, electric dipole, electric field due
to a dipole, torque on a dipole in uniform electric fleld.

(xxxxii)
Electric flux, statement of Gauss’s theorem and its applications to find field due to infinitely long
straight wire, uniformly charged infinite plane sheet and uniformly charged thin spherical shell (field
inside and outside).
Electric potential, potential difference, electric potential due to a point charge, a dipole and system of
charges; equipotential surfaces, electrical potential energy of a system of two point charges and of
electric dipole in an electrostatic field.
Conductors and insulators, free charges and bound charges inside a conductor. Dielectrics and electric
polarisation, capacitors and capacitance, combination of capacitors in series and in parallel, capacitance
of a parallel plate capacitor with and without dielectric medium between the plates, energy stored in
a capacitor. Van de Graaff generator.

Unit II: Current Electricity (Periods 22)


Electric current, flow of electric charges in a metallic conductor, drift velocity, mobility and their
relation with electric current; Ohm’s law, electrical resistance, V-I characteristics (linear and non-
linear), electrical energy and power, electrical resistivity and conductivity. Carbon resistors, colour
code for carbon resistors; series and parallel combinations of resistors; temperature dependence of
resistance.
Internal resistance of a cell, potential difference and emf of a cell,combination of cells in series and
in parallel.
Kirchhoff’s laws and simple applications. Wheatstone bridge, metre bridge.
Potentiometer - principle and its applications to measure potential difference and for comparing emf
of two cells; measurement of internal resistance of a cell.

Unit III: Magnetic Effects of Current and Magnetism (Periods 25)


Concept of magnetic field, Oersted’s experiment.
Biot - Savart law and its application to current carrying circular loop.
Ampere’s law and its applications to infinitely long straight wire. Straight and toroidal solenoids,
Force on a moving charge in uniform magnetic and electric fields. Cyclotron.
Force on a current-carrying conductor in a uniform magnetic field. Force between two parallel current-
carrying conductors-definition of ampere. Torque experienced by a current loop in uniform magnetic
field; moving coil galvanometer-its current sensitivity and conversion to ammeter and voltmeter.
Current loop as a magnetic dipole and its magnetic dipole moment. Magnetic dipole moment of a
revolving electron. Magnetic field intensity due to a magnetic dipole (bar magnet) along its axis and
perpendicular to its axis. Torque on a magnetic dipole (bar magnet) in a uniform magnetic field; bar
magnet as an equivalent solenoid, magnetic field lines; Earth’s magnetic field and magnetic elements.
Para-, dia- and ferro - magnetic substances, with examples. Electromagnets and factors affecting
their strengths. Permanent magnets.

(xxxxiii)
Unit IV: Electromagnetic Induction and Alternating Currents (Periods 20)
Electromagnetic induction; Faraday’s laws, induced emf and current; Lenz’s Law, Eddy currents.
Self and mutual induction.
Alternating currents, peak and rms value of alternating current/voltage; reactance and impedance;
LC oscillations (qualitative treatment only), LCR series circuit, resonance; power in AC circuits,
wattless current.
AC generator and transformer.

Unit V: Electromagnetic waves (Periods 4)


Need for displacement current, Electromagnetic waves and their characteristics (qualitative ideas
only). Transverse nature of electromagnetic waves.
Electromagnetic spectrum (radio waves, microwaves, infrared, visible, ultraviolet, X-rays, gamma
rays) including elementary facts about their uses.

Unit VI: Optics (Periods 30)


Reflection of light, spherical mirrors, mirror formula. Refraction of light, total internal reflection
and its applications, optical fibres, refraction at spherical surfaces, lenses, thin lens formula,
lensmaker’s formula. Magnification, power of a lens, combination of thin lenses in contact combination
of a lens and a mirror. Refraction and dispersion of light through a prism.
Scattering of light - blue colour of sky and reddish apprearance of the sun at sunrise and sunset.
Optical instruments : Human eye, image formation and accommodation correction of eye defects
(myopia, hypermetropia) using lenses. Microscopes and astronomical telescopes (reflecting and
refracting) and their magnifying powers.
Wave optics: Wave front and Huygen’s principle, relection and refraction of plane wave at a plane
surface using wave fronts. Proof of laws of reflection and refraction using Huygen’s principle.
Interference Young’s double slit experiment and expression for fringe width, coherent sources and
sustained interference of light. Diffraction due to a single slit, width of central maximum.
Resolving power of microscopes and astronomical telescopes. Polarisation, plane polarised light
Brewster’s law, uses of plane polarised light and Polaroids.

Unit VII: Dual Nature of Matter and Radiation (Periods 8)


Dual nature of radiation. Photoelectric effect, Hertz and Lenard’s observations; Einstein’s photoelectric
equation-particle nature of light.
Matter waves-wave nature of particles, de Broglie relation. Davisson-Germer experiment
(experimental details should be omitted; only conclusion should be explained).

(xxxxiv)
Unit VIII: Atoms & Nuclei (Periods 18)
Alpha-particle scattering experiment; Rutherford’s model of atom; Bohr model, energy levels,
hydrogen spectrum.
Composition and size of nucleus, atomic masses, isotopes, isobars; isotones. Radioactivityalpha,
beta and gamma particles/rays and their properties; radioactive decay law.
Mass-energy relation, mass defect; binding energy per nucleon and its variation with mass number;
nuclear fission, nuclear fusion.

Unit IX: Electronic Devices (Periods 18)


Energy bands in solids (Qualitative ideas only) conductors, insulator and semiconductors;
semiconductor diode – I-V characteristics in forward and reverse bias, diode as a rectifier; I-V
characteristics of LED, photodiode, solar cell, and Zener diode; Zener diode as a voltage regulator.
Junction transistor, transistor action, characteristics of a transistor, transistor as an amplifier (common
emitter configuration) and oscillator. Logic gates (OR, AND, NOT, NAND and NOR). Transistor as
a switch.

Unit X: Communication Systems (Periods 10)


Elements of a communication system (block diagram only); bandwidth of signals (speech, TV and
digital data); bandwidth of transmission medium. Propagation of electromagnetic waves in the
atmosphere, sky and space wave propagation. Need for modulation. Production and detection of an
amplitude-modulated wave.

Practicals
Every student will perform atleast 15 experiments (7 from section A and 8 from Section B) The
activities mentioned here should only be for the purpose of demonstration. One Project of three
marks is to be carried out by the students.

B. Evaluation Scheme for Practical Examination: Total Periods : 60

Two experiments one from each section 8+8 Marks

Practical record (experiments & activities) 6 Marks

Project 3 Marks

Viva on experiments & project 5 Marks

(xxxxv)
Total 30 Marks

SECTION A
Experiments
(Any 7 experiments out of the following to be performed by the students)
1. To find resistance of a given wire using metre bridge and hence determine the specific resistance
of its material
2. To determine resistance per cm of a given wire by plotting a graph of potential difference versus
current.
3. To verify the laws of combination (series/parallel) of resistances using a metre bridge.
4. To compare the emf of two given primary cells using potentiometer.
5. To determine the internal resistance of given primary cell using potentiometer.
6. To determine resistance of a galvanometer by half-deflection method and to find its figure of
merit.
7. To convert the given galvanometer (of known resistance and figure of merit) into an ammeter
and voltmeter of desired range and to verify the same.
8. To find the frequency of the a.c. mains with a sonometer.

Activities
1. To measure the resistance and impedance of an inductor with or without iron core.
2. To measure resistance, voltage (AC/DC), current (AC) and check continuity of a given circuit
using multimeter.
3. To assemble a household circuit comprising three bulbs, three (on/off) switches, a fuse and a
power source.
4. To assemble the components of a given electrical circuit.
5. To study the variation in potential drop with length of a wire for a steady current.
6. To draw the diagram of a given open circuit comprising at least a battery, resistor/rheostat, key,
ammeter and voltmeter. Mark the components that are not connected in proper order and correct
the circuit and also the circuit diagram.

(xxxxvi)
SECTION B
Experiments
(Any 8 experiments out of the following to be performed by the students)
1. To find the value of v for different values of u in case of a concave mirror and to find the focal
length.
2. To find the focal length of a convex mirror, using a convex lens.
3. To find the focal length of a convex lens by plotting graphs between u and v or between 1/u and
1/v.
4. To find the focal length of a concave lens, using a convex lens.
5. To determine angle of minimum deviation for a given prism by plotting a graph between angle of
incidence and angle of deviation.
6. To determine refractive index of a glass slab using a travelling microscope.
7. To find refractive index of a liquid by using (i) concave mirror, (ii) convex lens and plane mirror.
8. To draw the I-V characteristic curve of a p-n junction in forward bias and reverse bias.
9. To draw the characteristic curve of a zener diode and to determine its reverse break down voltage.
10. To study the characteristic of a common - emitter npn or pnp transistor and to find out the values
of current and voltage gains.
Activities (For the purpose of demonstration only)
1. To identify a diode, an LED, a transistor, and IC, a resistor and a capacitor from mixed collection
of such items.
2. Use of multimeter to (i) identify base of transistor (ii) distinguish between npn and pnp type
transistors (iii) see the unidirectional flow of current in case of a diode and an LED (iv) check
whether a given electronic component (e.g. diode, transistor or IC) is in working order.
3. To study effect of intensity of light (by varying distance of the source) on an L.D.R.
4. To observe refraction and lateral deviation of a beam of light incident obliquely on a glass slab.
5. To observe polarization of light using two Polaroids.
6. To observe diffraction of light due to a thin slit.
7. To study the nature and size of the image formed by (i) convex lens (ii) concave mirror, on a
screen by using a candle and a screen (for different distances of the candle from the lens/mirror).
8. To obtain a lens combination with the specified focal length by using two lenses from the given
set of lenses.

(xxxxvii)
SUGGESTED INVESTIGATORY PROJECTS
CLASS XII

1. To study various factors on which the internal resistance/emf of a cell depends.


2. To study the variations, in current flowing, in a circuit containing a LDR, because of a variation.
(a) in the power of the incandescent lamp, used to ‘illuminate’ the LDR. (Keeping all the lamps at a
fixed distance).
(b) in the distance of a incandescent lamp, (of fixed power), used to ‘illuminate’ the LDR.
3. To find the refractive indices of (a) water (b) oil (transparent) using a plane mirror, a equiconvex lens,
(made from a glass of known refractive index) and an adjustable object needle.
4. To design an appropriate logic gate combinatin for a given truth table.
5. To investigate the relation between the ratio of
(i) output and input voltage and
(ii) number of turns in the secondary coil and primary coil of a self designed transformer.
6. To investigate the dependence, of the angle of deviation, on the angle of incidence, using a hollow
prism filled, one by one, with different transparent fluids.
7. To estimate the charge induced on each one of the two identical styro foam (or pith) balls suspended
in a vertical plane by making use of Coulomb’s law.
8. To set up a common base transistor circuit and to study its input and output characteristic and to
calculate its current gain.
9. To study the factor, on which the self inductance, of a coil, depends, by observing the effect of this
coil, when put in series with a resistor/(bulb) in a circuit fed up by an a.c. source of adjustable frequency.
10. To construct a switch using a transistor and to draw the graph between the input and output voltage
and mark the cut-off, saturation and active regions.
11. To study the earth’s magnatic field using a tangent galvanometer.

Recommended Textbooks.
1. Physics, Class XI, Part -I & II,

(xxxxviii)
Heat and Thermodynamics

Learning Objectives:
● Understand the concept of heat and temperature
● Know about the specific heat capacity and latent heat and to apply the same in day to day
activities
● Know about thermal expansion and establish the relation between α, β and γ
● Solve the problem based on thermal equilibrium
● Understand the different mode of heat transfer and application based on the coefficient of
heat transfer (Thermal conductivity coefficient)
● Derive the expression for the work done in different thermodynamical process
● Interpret the P-V, P-T, V-T graphs
● Establish the relation between two different specific heat capacities for gases
● Know about the working of heat engine and refrigerator and their efficiency/COP
● Know the assumption of kinetic theory of gases and derive a relation
● For the pressure exerted by gas can the walls of the container
● Obtain different gas laws from the pressure expression
● Understand the concept of degree of freedom and equilibration law of energy

CONCEPTUAL PROBLEMS
1. What change would you expect in the space between AB, CD and EF it the metalize bodies are
uniformly heated?

A
B E F
C D

Hint. Space between AB and EF will increase while between CD will remain same.

1
Note:
During thermal expansion space between molecules increases.
2. If coefficients of linear expansion of a cube of side length ‘l’ be α 1, α 2 and α 3 along x-axis, y-axis
and z-axis respectively. Find the coefficient of superficial expansion and cubical expansion in
terms of α 1, α 2, and α 3.
y
Hint. Let change in length along x, y and z axes respectively be
∆lx = α1l
∆ly = α2l

∆lz = α3l

(when change in temperating is 1°C)


∴ Coefficient of superficial expansion β1, β2 and β3 for xy, yz and zx plans respectively can be

(l + α1l )(l + α 2l ) − l 2
given by β1 =
2l 2
l 2 + α1l 2 + α 2l 2 + α1α 2 l 2 − l 2
=
l2
= α1 + α2 (α1 and α2 are very small
∴ α1 α2 l2 can be neglacted)
Similarly, β2 = α2 + α3 and β3 = α3 + α1
Now coefficient of cubical expansion can be given as
(l + α1l )(l + α 2l )(l + α 3l ) − l 3
γ=
3l 3
= α1 + α2 + α3
If α1 = α2 = α3 = α(say) for isotropic meterials, then
β1 = β2 = β3 = 2α
and γ = 3α

Note:
For isotropic materials, thermal, electrical and optical properties are identical, but for anisotropic material
these are different in different directions.
3. Draw a plot between Fahrenheit temperature versus Celsius temperature. Also establish the
relations between the two scales.
Hint. The plot between Fahrenheit scale and Celsius scale is shown in figure.

2
As ice point have been assigned values 0°C and 32° F and steam point 100 °C and 212° F in Celsius
and Fahrenheit scales respectively.

Q F − 32 QC − 0 ok
Hence =
212 − 32 100 − 0 212

Q F − 32 QC ∆θF = 1 8 0°
or =
9 5
32
1 ∆θC = 1 00 °
or QC = (5Q F − 160)
9 0 1 0 0 °C

Note:
Difference of 1°C is not the same as the difference of 1°F, hence the lower point and upper point of scales
are important and the division of scales in also important.
4. 100 g ice is added to the 100 g water at 40°C. What will the final temperature.
Hint. The amount of heat required to melt the ice.
= mL
= 100 × 80 = 8000 Cal.
The maximum amount of energy water can loose = mc∆θ
= 100 × 1 × 40 = 4000 Cal
3 mc∆θ < mL
∴ Final temperature will remain zero degree Celsius.

Note:
During this thermodynamical process the amount of heat given to substance does not change the internal
energy of system and hence temperature does not increase. As per the first law of thermodynamic
dQ = dU + PdV
\ dU = 0
∴ dQ = PdV
5. Show that a gas does less work in adiabatic expansion as that in an isothermal expansion.
Hint. Let a gas be at the point P corresponding to P0, V0, T0. When it is allowed to expand isothermally the
corresponding curve is AB.
When it is allowed to do so adiabatically, the corresponding curve is CD. Let the gas expand so that
new value is V.
Under isothermal expansion the pressure and temperature are P1 and T1 = T2 respectively and under
adiabatic expansion these are P2 and T2, then

P1V P0 V0
=
T0 T0

3
P0 V0 P
∴ P1 = A C
V
P2 V P0 V0
and =
T2 T0
P
P0 B
P0 V0T2
∴ P2 =
V T0
D
T2 0 V0 V
Hence P2 = P1
T0 V

3 T2 < T0 in adiabatic expansion, therefore P2 < P1 hence the adiabatic curve drops below the
isothermal curve. For a similar line of represents for adabatic expansion P2 > P1 owing to the raise in
temperature and the adiabatic curve in above the isothermal curve.

P0

Iso the rm a l A d iab atic

Iso the rm a l
A d iab atic P0

V0 V V V0
(expansion) (compression)
Since the area below the P-V curve, gives work done, so the gas does less work in adiabatic expan-
sion than in the isothermal expansion.

Note:

CP
The P-V graph in γ = C times more steeper in case of adiabatic thermodynamical process.
V

Work done in thermodynamical process can be calculated by calculating the area under P-V curve.
6. A metre long narrow above held horizontally (and closed at one end) contains a 78 cm long
mercury thread which traps a 15 cm column of air. What happens if the tube in held vertically
with open end at the bottom?
Hint. When the tube in horizontal the air of volume 15 cm long column is trapped. In this situation this air
coloumn is at atmospheric pressure and when tube is vertical with length 100 cm, pressure of air is
reduced to height h of mercury column and volume of air is the volume of (100 – 76th = (24 + h)
height of column.
In this situation some mercury will flow out of the tube to balance with the atmospheric pressure of
76 cm of mercury column.

4
At constant temperature
P1V1 = P2V2
76 (15 A) = h (24 + h) A
(A = Area of the cross - section of the tube)
⇒ h = 24 cm

Note:
To maintain the equilibrium at atmospheric pressure outside the mercury of 24 cm length of tube flows out
and the mercury of length 76 – 24 = 52 cm and air of length 100 – 52 = 48 cm will remain in the tube in
vertical position.

Activities
I. Let the time taken in melting the ice in case A and B be t1 and t2. In case A largely heat is transferred
through convection while in case B through conduction. In case of A rate of heat transfer.

Q (θ1 − θ 2 )
= k 1A
t1 l

θ2
Q k 2 A(θ 2 − θ1 )
In case B, = Q1 →
t2 l
For same value of Q, A and l A
B
1 1
= k1 (θ 2 − θ1 ) or k1 =
t1 t 2 (θ 2 − θ1 )
QV →
1 1 θ2 θ1
= k 2 (θ 2 − θ1 ) or k 2 = t (θ − θ )
t2 2 2 1

In case A (θ2 – θ1) is large and t1 is small while in case B (θ2 – θ1) is small and t2 is large but the
product of t1(θ2 – θ1) and t2 (θ2 – θ1) in both the cases are same. Thus, it can be concluded that the
coefficient of rate of heat transfer in conduction and convection is same.

II. Inflate a balloon and bring it on the candle flame. Now


take another balloon half filled with water and bring on
the candle flame. Write your observations and three point
of conclusion. Try to relate your conclusion with boiling
point, ignition temperature, pressure and volume.

5
III. Take bottle of glass close the lid and heat to rise its temperature by 25°C by supplying heat with
constant rate and note down time. Now open the lid and put a rubber balloon at its mouth (air tight)
and rise the temperature by 25°C in the same way and note down the time. Explain your observations
related to the CP and CV and less heat is needed to rise the temperature at constant volume.
 3
IV. Take a graduated jar filled with water   at room temperature and immersed in it a inflated balloon.
4
Now replace the water by water at 0°C (chilled water) and again immersed the same balloon in it.
Find the change in the level of water in both the cases and try to relate your findings with change in
temperature and change in volume and explain on the basis of Law of thermodynamics.

1. The coefficient of volume expansion of glycerine is 49 × 10–5 k–1. What is the fractional change
in the density for 30° C rise in temperature?
Sol. 3 ∆V = γV∆θ
∆V
∴ = γ∆θ
V
∆ρ ∆V
⇒ = − = −γ∆θ
ρ V
−5
= −49 × 10 × 30 = −0.0147
= −1.5 × 10−2
(–ve sign indicates the density is decreasing)
2. A child is running a temperature of 101°F is given an antipyrin (i.e., a medicine that lowers
fever) which causes an increase in the rate of evaporation of sweat from the body. If the fever is
brought down to 98°F in 20 min, what is the average rate of extra evaporation caused, by the
drug. Assume the evaporation mechanism to be the only way by which heat is lost. The mass of
the child is 30 kg. The specific heat of human body is approximately the same as heat of water,
and latent heat of evaporation of water at that temperature is about 580 cal g–1.
Sol. Let m and m1 be the mass of child and evaported water, then
mc∆θ = m1L

mc∆θ 5  −5 
or m1 = = 30 × 1000 ×  101°F − 98°F − °C
L 3 3
= 86.2 g
86.2g
Rate of evaporation = = 4.91 g min −1
20 min
3. A body cools 80°C to 50°C in 5 minutes. Calculate the time it takes to cool from 60°C to 31°C.
The temperature of the surrounding is 20°C.

6
Sol. Average temperature of the liquid
80 + 50
= = 65°C
2
Excess of temperature = 65 – 20 = 45°C
Rate of fall of temperature,
dθ1 (80 − 50)
= − = −6°C per min .
dt 5 min
⇒ – 6 dt = k 45 (k is constant)
For next step :
Average temperature of the liquid
60 + 30
= = 45°C
2
Excess of temperature = (45 – 20) = 25°C
Rate of fall in temperature,
dθ 2 (60 − 30) 30°C
= − =−
dt t min t min

30°
Again − = k(25)
t min

6t k(45) 9
⇒ = = 5
30 k(25)

30 × 9
t= = 9 min .
30
4. A cylinder with a movable piston contains 3 moles of hydrogen at standard temperature and
pressure. The walls of the cylinder are made of a heat insulator, and the piston is insulated by
having pile of sand on it. By what factor does the pressure of the gas increase if the gas is
compressured to half of its original volume?
Sol. For adiabatic process
P1V1γ = P2 V2γ
γ
V 
or P2 = P1  1 
 V2 
γ
V 
= 1 1 
 V2 
= (2)1.4 = 2.64 atm

7
5. A thermodynamic system is taken from an original state to an intermediate state by the linear
process as shown in figure. Its volume is then reduced to the original value from E to F by an
isobaric process. Calculate the total work done by the gas from D to E to F.
Sol. Work done = area of cycle DEF 2
(n m )
D
1 600
= (EF)(FD)
2

1 300 E
= (5.0 − 2.0)(600 − 300) P F
2
= 450 J 0 20° 500
3

As the process FD is isochoric ∴ work done in it is zero. V (m )

6. A car tyre contains air at a pressure 4 atm and its temperature is 27° C. The tyre suddenly
bursts. Calculate the resulting temperature (γγ = 1.4)
1−γ γ
Sol. 3 P11−γ T1γ = P2 T
1−γ
P γ
∴ T2 = T1  1 
P 
2
1.4 −1
 1  1.4
= 300  
 4
= 201.9 K.
7. From a certain apparatus, the diffusion rate of hydrogen has an average value of 28.7 cm3 s–1.
The diffusion of another gas under the same conditions is measured to have an average rate of
7.2 cm3 s–1. Identify the gas.

3RT 1
Sol. 3 v= ⇒ vα
M M

vH M 28.7 M
∴ = ⇒ =
v MH 7.2 MH

2
 28.7 
M = MH  = 16M H
 7.2 
or

Thus, the second gas its molar mass equal to 16 times the molar mass of hydrogen i.e., the gas is
oxygen.
8. A gas in equilibrium has uniform density and pressure through its volume. This is strictly true
only if there are no external influences. A gas column under gravity, for example, does not have
uniform density (and pressure). As you might expect, its density decreases with height. The
precise dependence is given by the so called law of atmospheres.

8
 –mg(h 2 – h1 ) 
n2 = n1 exp  
 k BT 
where n2 and n1 refere to number density at height h2 and h1 respectively. Use this relation to
derive equation for sedimentation equilibrium of a specimen in a liquid column.

 –mgNA(ñ – ñ1 )(h 2 – h1 ) 
n2 = n1 exp  
 ñRT 
where ρ is the density g the suspanded particles and ρ 1 that of surroundign medium [NA is
Avogadro’s number, and R is universal gas constant].
Sol. Applying Archimedes principle, decrease in pressure due to increase in height under granting
dρ = –ρgdh (ρ, g are constant)
P = nkBT
dP = KBT dh (T = constant)

dn mg N A mg dh
or = − k T dh = R . T
n B

h2 h2
dn − N A mg
∫ n = RT ∫ dh
h1 h1

n  N mg
log  2  = − A (h 2 − h1 )
 n1  RT

 N mg(h 2 − h1 ) 
or n = n1exp  − A 
 RT
But effective mass of the practice
4 3 m
= πr (ρ − ρ1 ) = (ρ − ρ1 )
3 g

 ρ− 
= m 1 − 
 ρ

 N A mg  ρ1  
∴ n2 = n1exp  −  1 − (h − h ) 
ρ 
2 1
 RT  
9. Estimate the average thermal energy of a helium atom at (i) room temperature (27°C) (ii) the
temperature on the surface of the sun 6000 K, (iii) the temperature of 10 million kelvin (the
typical core temperature in the case of a star)

9
3
Sol. (i) E= k BT
2

3
= × 1.38 × 10−23 × (27 + 273)
2
= 6.21 × 10 −23 J

3
(ii) E= k BT
2

3
= × 1.38 × 10 −23 × 6000
2
= 1.24 × 1019 J

3
(iii) E= k BT
2

3
= × 1.38 × 10 −23 × 10 × 106
2
= 2.1 × 10 −16 J
10. A cylinder of fixed capacity 44.8 litres contains helium gas at standard temperature and pressure.
What is the amount of heat needed to rise the temperature of the gas in the cylinder by 15.0°C?
Sol. Number of moles in cylinder
44.8
n= =2
22.4
Amount of heat required at constant
volume, ∆Q = ∆U = nCV ∆θ

3 
= n  R  ∆θ
2 

3
= 2× × 8.31 × 15
2
= 374 J.

10
Oscillations and Waves

Learning Objectives :
● Periodic and oscillatory motions
● Simple harmonic motion and uniform circular motion
● Velocity and acceleration in simple harmonic motion
● Force law for simple harmonic motion
● Energy in simple harmonic motion
● Some systems executing SHM
● Damped simple harmonic motion
● Forced oscillations and resonance
● Transverse and Longitudinal waves
● Displacement relation in a progressive wave
● The speed of a travelling wave
● The principle of superposition of waves
● Reflection of waves
● Beats
● Doppler effect

CONCEPTUAL PROBLEMS

1. A girl is sitting on a swing. Another girl sits by her side. What will be the affects on the periodic time
of the swing?
Hint. There will be no effect on the time period of oscillation. The time period of swing depends on the
length of the swing and the acceleration due to gravity at the place. If another girls sits only mass of
the swing changes and time period doesn’t depend on mass.
2. The girl sitting on a swing stands up. How will the time period be affected?
Hint. If the girl stands up the distance between point of suspension and center of mass of the girl will
decrease thus decreasing the length of the pendulum. Thus, time period of the swing will also decrease.
3. What provides restoring force for SHM in [a] pendulum [b] spring pendulum and [c] liquid oscillating
in U tube?

11
Hint. In Simple pendulum gravity provides the restoring force where as in spring pendulum the elasticity
of the spring provides the restoring force. For U tube the weight of the liquid provides restoring
force.
4. If pendulum clock is taken to mountain top does it loose or gain time, assuming it gives correct time
at lower elevation?
Hint. As we move above the surface of earth the acceleration due to gravity is going to decrease thus the
time period of oscillation of the pendulum will increase. Thus the clock will go slow on the mountain
top and it looses time.
5. The amplitude of simple harmonic oscillator is doubled how will it affect the [a] the maximum
velocity [b] the total energy [c] the period of oscillation?
Hint. The maximum velocity V = rw, thus if the amplitude is doubled the maximum velocity becomes
double. The total energy is directly proportional to the square of amplitude of the wave. Thus, if
amplitude is doubled the total energy becomes four times. The time period is independent of the
amplitude of motion and remains unchanged.
6. At what distance from the mean position the kinetic and potential energy of simple harmonic oscillator
equal?
Hint. Kinetic energy = potential energy
y =r 2
7. For an oscillating pendulum, is tension in the string constant throughout? If not, where it is [a]
minimum and [b] maximum?
Hint. tension in the string changes with the change in angle which the pendulum string makes with the
vertical. It is maximum at the lowest point and minimum at the extreme position.
8. Why amplitude of vibrating pendulum should be small?
Hint. If the amplitude of vibration is small we can make an assumption that mg sinq = mgq, but if the
amplitude of oscillation is large we can’t make this assumption thus the restoring acceleration will
not be proportional to the displacement and motion will not be SHM.
9. What is necessary and sufficient condition for motion to be simple harmonic?
Hint. The necessary and sufficient condition for motion to be SHM is that the restoring force is always
proportional to the displacement and the direction of restoring force is opposite to displacement.
10. When a body of mass 2kg is suspended by a spring the spring is stretched. If the body is pulled down
and released, it oscillates up and down. What force should be applied on the body by the spring when
it passes through the mean position?
Hint. When body passes through the mean position, the weight of the body acting downward is balanced
by upward restoring force in the spring. Thus net force as well as the acceleration at the mean
position is zero.
11. Will the time period of spring pendulum change if it is taken to moon?

12
Hint. The time period of spring pendulum depends on the mass of the body attached and spring constant of
the spring. Both the values will remain same on moon as on earth thus time period will remain
unchanged.
12. A vibrating simple pendulum of time period T is a placed in a lift, which is accelerating downwards.
What will be the effect on the time period?
Hint. If the lift accelerates downward, pseudo force acts on the body in upward direction. Thus the value
of acceleration becomes (g-a) and the time period is going to increase as effective value of g decreases.
13. What are the factors on which the natural frequency of the body depends?
Hint. Natural frequency of body depends upon [a] elastic properties of the materials of the body and [b]
dimensions of the body.
14. Is the damping forces acting on the body in SHM constant?
Hint. No the damping forces are dependent on the velocity of the body therefore they can’t be constant as
in SHM velocity keeps on changing with time.
15. The bob of simple pendulum is made of ice. How will the time period change when the ice melts?
Hint. If the ice melts, the time period will remain unchanged because the distance between point of
suspension and center of the bob remains unchanged. But if the center of gravity of ice starts moving
upwards due to melting of ice then the time period will decrease.
16. What would happen to the motion of oscillating system if the sign of force term in equation F=-kx is
changed?
Hint. If we change the sign equation will be F=kx which implies that force will be in the direction of the
displacement. Thus motion will not be SHM but it will be linearly accelerated motion.
17. The bob of simple pendulum is a ball full of water. If a fine hole is made at the bottom so that water
slowly leaks, what will be the effect on the time period of the pendulum?
Hint. If a small hole is made at the bottom of the ball, and water starts coming out the center of mass of the
pendulum moves down. Thus the effective length of the pendulum increases and time period also
goes on increasing. But when the ball is completely empty the center of mass again shifts to initial
value and time period will also return to its initial lesser value.
18. The maximum acceleration of a simple pendulum is a while maximum velocity is v. what is the
displacement amplitude?
Hint. The maximum acceleration of the pendulum is a = rω2 and maximum velocity is v = rω. From these
two equations we can say that r = v2/a

WAVES

1. Frequency is the most fundamental property of a wave why?


Hint. When a wave travels from one medium to other its wavelength as well as velocity may change, but
frequency does not change. Hence frequency is the fundamental property of a wave.

13
2. Name two properties of a material medium responsible for the propagation of waves through it.
Hint. Property of elasticity and insert.
3. How is it possible to detect the approaching of a distant train by placing the ear close to the railway
line?
Hint. Sound waves travel much faster in solids than that in air. Further, due to high elasticity of solids,
sound waves do not die out in solids as soon as in air.
4. Ocean waves hitting a beach are always found to be nearly normal to the shore. Why.
Hint. Ocean waves are transverse in nature and spread out in the form of concentric circles. When these
waves reach the beach shore, their radius of curvature becomes so large that they can be treated as
plane waves. Hence the ocean waves hit the beach nearly normal to the shore.
5. Does the sound of a bomb explosion travel faster than the sound produced by a humming bee.
Hint. Velocity of sound does not depend on its loudness so though the two sounds have entirely different
characteristics they travel with the same speed.
6. Sometimes, in a stringed instrument, a thick wire is wrapped by a thin wire. Why?
Hint. This increases mass per unit length and hence helps in obtaining a desired low frequency which
would otherwise require a string of inconveniently large length.
7. Why is note produced by a open organ pipe sweeter than that produced by the closed organ pipe.
Hint. The note produced by open organ pipe has both odd and even harmonics white the closed organ pipe
has only odd harmonics. Due to the presence of large number of overtones or harmonics the note
produced by open organ pipe is more pleasant.
8. Why are there so many holes in a flute.
Hint. The flute is an open organ pipe. The location of open and can be changed by keeping the one hole
open and closing the other holes, thus the frequency of note produced can be changed.
9. How does the frequency of a vibrating wire change, when the attached load in immersed in water?

Hint. When the load is immersed in water, apparent weight observes so tension decreases. A 2 T , the
frequency of vibrating wire decreases.
10. The beats are not heard if the difference in frequencies of the two sounding notes is more than 10.
Why?
Hint. As the number of beats will be more than 10, our ear is not able to distinguish between two sounds as
th
 1
separation the time interval betwen them is less than   of a second.
 10 
11. (a) What physical change occur when a source os sound moves and the listener is stationary.
Hint. (a) Wavelength of sound wave changes
12. What physical change occurs when the source of sound is stationary and the listener moves.
Hint. The number of sound waves received by the listener changes.

14
13. Two progressive sound waves each of frequency 170 Hz and travelling in apposite direction superpose
to produce stationary waves. The speed of sound in air is 340 ms–1. What is the separation between (i)
two successive nodes, (ii) a node and its nearest antinode.
v 340
Hint. λ = = = 2m
λ 170
λ
distance betwee two nodes or antinodes = = 1m
2
λ
distance between a node nearest antinode = = 0.5m
4
14. An open pipe is in second harmonic with frequency f1 now one end of the tube is closed and frequency
is increased to f2 such that the resonance occurs again in nth harmonic. Find value of n. How are f1 &
f2 related?
v v
Hint. f1 = 2 =
2L L
nv
fn = where n is an odd integete
4l
fn will be just greater than f when n = 5
5v 5
Hence f2 = = f1
4L 4

ACTIVITIES
I. Take three simple pendulums of the same length and bobs of same diameter but different masses,
determine their Time Periods. Measure the time periods for different amplitudes.

Answer: All the three pendulums will have the same time period as it does not depend on the mass of the
bob. For angular displacement as large as 20° sine of the angle is equal to the angle and it can be seen that
for 50° amplitude the error is only 5% .Though we prefer angle of 20° but 50° can be used also.

15
II. Take three balls of same size and suspend them with thread of same length, to form five pendulums.
Keep one empty and fill the others with different level of water. Determine the time period of the
pendulums .Will it be same. Justify.

Answer: The time periods will be different as the water will also execute oscillatory motion.
III. Take a small ball fill it with sand and suspend it with a thread. Determine the time period and then
make a small hole and determine the time period of the pendulum for different quantity of sand in the
ball

Answer: The time period increases initially as effective length increases and the it will decrease
when the ball is empty, the first and last case will have same time period.
IV. Resonance experiment can be done effectively using a buzzer of lkhz instead of tuning fork.

16
V. Demonstration of wave motion using cellophene tape, straw and beads.
Make the wave motion apparatus as shown. Produce a disturbance in one straw and see the wave
travel. We can show that the wave velocity does not depend on the amplitude of the wave.
Increase the weight of the beads and see

VI. Standing waves using rubber band.

VII.
Demonstration
The diagram below illustrates constructive interference. Students A and B hold opposite ends of a coil
spring toy or similar spring. At the same time, they generate pulses large enough to just clear the bricks.
When the waves meet, they constructively interfere and produce a larger wave that knocks over the pop
can. The waves then continue to travel past the bricks without touching. With a little practice, this can be
accomplished with regularity. Videotape and play back the tape in slow motion.

17
VIII.
Sprinkle some fine dust on a fixed metal plate and stroke the plate with a violin bow. The dust will move
away from certain areas and toward others. The particles move away from the interference nodes. The dust
formation is called a Chladni pattern.

IX.
The students’ understanding will be increased if they experience wave phenomena through a series of
laboratory exercises. Waves in one dimension can be investigated with strings, springs, and/or a coil spring
toy like a Slinky. A coil spring toy is quite useful in demonstrating wave phenomena. For convenience, the
toy can be suspended from the ceiling or a support system (see diagram).

X.
Following the Exploration Activity
a. Evaluate student presentation of their findings and how they support them, as well their participation
in the discussion.
b. Have students build a concept map after the discussion of the exploration activity. A second map will
be built after completion of the unit and a comparison made to determine any changes in student
conceptions. Possible concepts to be included in the map are: restoring force, oscillation, simple
harmonic motion, period, frequency, amplitude, mass, acceleration, position, velocity and equilibrium.
One possible “expert” map is included below as an example. In concept mapping there is NO one
right answer.

18
O sc illatio n M o tio n

O c cu rs w ith a
H a s th e

R esto rin g F o rc e C h ara cteristics

W h ich is of
lin e arly
of
p ro po rtio n al
to F re qu e n cy
of
is
D isp la ce m e n t A m plitu d e

fro
F rommF rom W h ich de p en d s on
P e rio d

E q u ilib riu m
W h ich
d ep e n ds o n

P ro p ertie s
o f th e

P h y sica l S y ste m s

e xa m p les o f w h ic h a re

M a ss o n a
P e n du lu m Vib rating B a r
S p rin g

Question 1 :
A particle is in linear simple harmonic motion between two points, A and B, 10 cm apart. Take the direction
from A to 8 as the positive direction and give the signs of velocity, acceleration and force on the particle
when it is
(a) at the end A, (c) at the mid-point of AB going towards A,
(b) at the end B, (d) at 2 cm away from B going towards A,

19
(e) at 3 cm away from A going towards B, and
(f) at 4 cm away from B going towards A.
Answer :
(a) Zero, Positive, Positive
(b) Zero, Negative, Negative
(c) Negative, Zero, Zero
(d) Negative, Negative, Negative
(e) Zero, Positive, Positive
(f) Negative, Negative, Negative

Explanation:
The given situation is shown in the following figure. Points A and B are the two end points, with
AB = 10 cm. O is the midpoint of the path.
A O B
A particle is in linear simple harmonic motion between the end points
(a) At the extreme point A, the particle is at rest momentarily. Hence, its velocity is zero at this point.
Its acceleration is positive as it is directed along AO.
Force is also positive in this case as the particle is directed rightward.
(b) At the extreme point B, the particle is at rest momentarily Hence, its velocity is zero at this point.
Its acceleration is negative as it is directed along B.
Force is also negative in this case as the particle is directed leftward.
2 cm
(c)
A O B
The particle is executing a simple harmonic motion. O is the mean position of the particle. Its velocity
at the mean position 0 is the maximum. The value for velocity is negative as the particle is directed
leftward. The acceleration and force of a particle executing SHM is zero at the mean position,
2 cm
(d)
A O B
The particle is moving toward point 0 from the end B. This direction of motion is opposite to the
conventional positive direction, which is from A to B. Hence, the particle’s velocity and acceleration,
and the force on it are all negative,
3 cm
(e)
A D O B
The particle is moving toward point 0 from the end A. This direction of motion is from A to B, which
is the conventional positive direction. Hence, the values for velocity, acceleration, and force are all
positive.

20
4 cm
(f)
A O E B

This case is similar to the one given in (d).


Question 2 :
The motion of a particle executing simple harmonic motion is described by the displacement function,
x (t) = A cos (ωt + φ).
If the initial (r = 0) position of the particle is 1 cm and its initial velocity is ω cm/s, what are its
amplitude and initial phase angle? The angular frequency of the particle is n s–1. If instead of the cosine
function, we choose the sine function to describe the SHM: x = B sin (ωt + a), what are the amplitude and
initial phase of the particle with the above initial conditions.
Answer
Initially, at t = 0 :
Displacement, x = 1 com
Intial velocity, v = ω cm/sec.
Angular frequency, ω = rad/s–1
It is given that :
x(t) = A cos(ωt + φ)
A cos φ = 1 ...(i)
dx
Velocity, v=
dt
ω = − Aω sin(ωt + φ)
I = − A sin(ω × 0 + φ) = − A sin φ
A sin φ = –1 ...(ii)
Squaring and adding equations (i) and (ii), we get :
A 2 (sin 2 φ + cos 2 φ) = 1 + 1
A2 = 2
∴ A= 2 cm
Dividing equation (ii) by equation (i), we get :
tan φ = –1
3π 7 π
∴ φ= , ,...
4 4
SHM is given as :
x = B sin (ωt + a)
Putting the given values in this equation, we get :

21
I = B sin[ω × 0 + α)
B sin α = 1 ...(iii)
v = ωB cos (ωt + α)
Velocity,
Substituting the given values, we get :
π = πB sin α
B sin α = 1 ...(iv)
Squaring and adding equations (iii) and (iv), we get :
I = B sin[ω × 0 + α]
B sin α = 1 ...(iii)
v = ωB cos (ωαt + α)
Velocity,
Substituting the given values, we get :
π = πB sin α
B sin α = 1 ...(iv)
Squaring and adding equations (iii) and (iv), we get :
Squaring and adding equations (iii) and (iv), we get :
B2 [sin 2 α + cos 2 α ] = 1 + 1
B2 = 2
∴ B= 2 cm
Dividing equation (iii) by equation (iv), we get :
Bsin α 1
=
B cos α 1
π
tan α = tan
4
π 4π
∴ α= , ,...
4 4
Question 3 : Figure a and b correspond to two circular motions. The radius of the circle, the period revolution,
the initial position, and the sense of revolution (i.e. clockwise or an clockwise) are indicated on each figure.

22
Obtain the corresponding simple harmonic motions of the x-projection of the radi vector of the revolving
particle P, in each case.
Answer : (a) Time period, T = 2 s
Amplitude, A = 3 cm
π
At time, t = 0, the radius vector OP makes an angle with the positive x-axis, i.e. phase angle
2
π
φ=+ .
2
Therefore, the equation of simple harmonic motion for the x-projection of OP, at time is given by the
displacement equation:
 2πt 
x = A cos  + φ
 T 
 2πt π   2πt 
= 3cos  +  = −3sin 
 2 2  2 
∴ x = −3sin πt cm
(b) Time period, T = 4 s
Amplitude, a = 2 m
At time t = 0, OP makes an angle n with the x-axis, in the anticlockwise directic
Hence, phase angle, φ = + n
Therefore, the equation of simple harmonic motion for the x-projection of OP, at time is given as:

 2 πt   2πt 
x = a cos  + φ = 2 cos  + π
 T   4 

π 
∴ x = −2 cos  t  m
2 
Question 4 :
Plot the corresponding reference circle for each of the following simple harmor motions. Indicate the
initial (t = 0) position of the particle, the radius of the circle, a the angular speed of the rotating particle. For
simplicity, the sense of rotation may fixed to be anticlockwise in every case: (x is in cm and t is in s).
(a) x = –2 sin (3t + π/3)
(b) x = cos (π/6 – r)
(c) x = 3 sin (2πt + π/4)
(d) x = 2 cos πt
Answer
 π  π π
x = −2sin  3t +  = +2 cos  3t + + 
3 3 2
 5π 
= 2 cos  3t + 
6

23
 2π 
If this equation is compared with the standard SHM equation x = A cos  t + φ we get :
 T  th
Amplitude, A = 2 cm

Phase angle, φ = = 150°
6


angular velocity, ω = = 3 rad / sec.
T
The motion of the particle can be plotted as shown in the following figure.

π   π
(b) x = cos  − t  = cos  t − 
6   6
 2π 
If this equation is compared with the standard SHM equation x = A cos  t + φ we get :
 T  th
Amplitude, A = 1
π
Phase angle, φ = − = −30°
6

Angular velocity, ω = = 1 rad/s
T
The motion of the particle can be plotted as shown in the following figure.

24
 π
(c) x = 3sin  2πt + 
 4
 π π   3π 
= −3cos  2πt +  +  = −3cos  2πt + 
 4 2   4
 2π 
If this equation is compared with the standard SHM equation x = A cos  t + φ we get :
 T  th
Amplitude, A = 3 cm

Phase angle, φ = = 135°
4


Angular velocity, ω = = 2π rad / s
T
The motion of the particle can be plotted as shown in the following figure.

(d) x = 2 cos πt
 2π 
If this equation is compared with the standard SHM equation x = A cos  t + φ we get :
 T  th
Amplitude, A = 2 cm
Phase angle, φ = 0
Angular velocity, ω = π rad/s
The motion of the particle can be plotted as shown in the following figure.

25
Question 5 : Which of the following relationships between the acceleration a and the displacement of a
particle involve simple harmonic motion?
(a) a = 0.7x
(b) a = –200x2
(c) a = –l0x
(d) a = 100x3
Answer :
(c) A motion represents simple harmonic motion if it is governed by the force law:
F = –kx
ma = –k
k
∴ a=− x
m
Where,
F is the force
m is the mass (a constant for a body)
x is the displacement
a is the acceleration
k is a constant
Among the given equations, only equation a = –10 x is written in the above form w
k
= 10. Hence, this relation represents SHM
m
Question 6 : A steel wire has a length of 12.0 m and a mass of 2.10 kg. What should be the tension
on the wire so that speed of a transverse wave on the wire equals the speed of sound in dry air at
20 °C = 343 m s–1.
Answer :
Length of the steel wire, I = 12 m
Mass of the steel wire, m = 2.10 kg
Velocity of the transverse wave, V = 343 m/s
m 2.10
Mass per unit length, µ = = = 0.175kg m −1
l 12
For tension T, velocity of the transverse wave can be obtained using the relation:

T
v=
µ
∴ T = v2 µ
= (343)2 × 0.175=20588.575 ≈ 2.06 × 104N

26
Question 7 :

γP
Use the formula v = to explain why the speed of sound in air
ρ
(a) is independent of pressure
(b) increases with temperature
(c) increases with humidity
Answer
(a) Take the relation :

γP
v= ...(i)
ρ
Where,
Mass M
Density, ρ = =
Volume V
M = Molecular weight of the gas
V = Volume of the gas
Hence, equation (i) reduces to:

γ PV
v= ...(ii)
M
Now from the ideal gas equation for n = 1:
PV = RT
For constant T, PV = Constant
Since both M and y are constants, v = Constant
Hence, at a constant temperature, the speed of sound in a gaseous medium is independent of the
change in the pressure of the gas.
(b) Take the relation:
γP
v= ...(i)
ρ
For one mole of an ideal gas, the gas equation can be written as: PV = RT
PV = RT
RT
P= ...(ii)
V
Substituting equation (ii) in equation (i), we get:

γ RT γ RT
v= = ...(iv)
Vρ M

27
Where,
Mass, M = ρV is a constant
Y and P, are also constants
We conclude from equation (iv) that v ∝ T .
Hence, the speed of sound in a gas is directly proportional to the square root of the temperature of the
gaseous medium, i.e., the speed of the sound increases with an increase In the temperature of the
gaseous medium and vice versa.
(c) Let Vm and Vd be the speeds of sound in moist air and dry air respectively.
Let ρm and ρd be the densities of moist air and dry air respectively. Take the relation:

γP
v=
ρ
Hence, the speed of sound in moist air is:

γP
vm = ρm ...(i)

And the speed of sound in dry air is:

γP
vd = ρd ...(ii)

On dividing equations (i) and (ii), we get:

vm γ P ρd ρd
× =
vd = ρm γ P ρm

However, the presence of water vapour reduces the density of air, i.e.
ρd < ρm
∴ vm > vd
Hence, the speed of sound in moist air is greater than it is in dry air. Thus, in a gaseous medium, the
speed of sound increases with humidity.
Question 8:
A transverse harmonic wave on a string is described by

 π
y(x, t) = 3.0sin  36t + 0.018x + 
 4
Where x and y are in cm and c in s. The positive direction of x is from left to right.
(a) Is this a travelling wave or a stationary wave?
If it is travelling, what are the speed and direction of its propagation?

28
(b) What are its amplitude and frequency?
(c) What is the initial phase at the origin?
(d) What is the least distance between two successive crests in the wave?
Answer
(a) Yes; Speed = 20 m/s, Direction = Right to left
(b) 3 cm; 5.73 Hz
π
(c)
4
(d) 3.49 m
Explanation:
(a) The equation of a progressive wave travelling from right to left is given by the displacement function:
y (x, t) = a sin(ωt + kx + φ) ...(i)
The given equation is:
 π
y(x, t) = 3.0sin  36t + 0.018x +  ...(ii)
 4
On comparing both the equations, we find that equation (ii) represents a travelling wave, propagating
from right to left.
Now, using equations (i) and (ii) we can write:
ω = 36 rad/s and k = 0.018 m–1
We know that:
ω 2π
v= and λ =
2π k
Also,
v = vλ
 ω   2π  ω
∴ v =   ×   =
2π k k
36
= = 2000 cm/s = 20 m/s
0.018
Hence, the speed of the given travelling wave is 20 m/s.
(b) Amplitude of the given wave, a = 3 cm
Frequency of the given wave:
ω 36
v= = = 5.73 Hz
2π 2 × 3.14
π
(c) On comparing equations (i) and (ii), we find that the initial phase angle, φ =
4
(d) The distance between two successive crests or troughs is equal to the wavelength the wave.

29
Wavelength is given by the relation:

k=
λ
2π 2 × 3.14
∴ λ= = = 348.89 cm = 3.49 m
k 0.018
Question 9 : A hospital uses an ultrasonic scanner to locate tumours in a tissue. What is the wavelength of
sound in the tissue in which the speed of sound is 1.7 km s–1? The operating frequency of the scanner is
4.2 MHz.
Answer : Speed of sound in the tissue, v = 1.7 km/s = 1.7 × 103 m/s
Operating frequency of the scanner, v = 4.2 MHz = 4.2 x 106 Hz
The wavelength of sound in the tissue is given as:
v
λ=
V
1.7 × 103
= = 4.1 × 10 −4 m
4.3 × 10 6

Question 10 : A metre-long tube open at one end, with a movable piston at the other end, shows resonance
with a fixed frequency source (a tuning fork of frequency 340 Hz) when the tube length is 25.5 cm or
79.3 cm. Estimate the speed of sound in air at the temperature of the experiment. The edge effects may be
neglected.
Answer : Frequency of the turning fork, v = 340 Hz
Since the given pipe is attached with a piston at one end, it will behave as a pipe with one end closed
and the other end open, as shown in the given figure.

Such a system produces odd harmonics. The fundamental note in a closed pipe is given by the
relation:
λ
I1 =
4
Where,
Length of the pipe, I1 = 25.5 cm = 0.255 m
∴ λ = 4I1 = 4 × 0.255 = 1.02 m
The speed of sound is given by the relation:
v = vλ = 340 × 1.02 = 346.8 m/s

30
Question 11: A pipe 20 cm long is closed at one end. Which harmonic mode of the pipe is resonantly
excited by a 430 Hz source? Will the same source be in resonance with the pipe if both ends are open?
(Speed of sound in air is 340 m s–1).
Answer : First (Fundamental); No
Length of the pipe, I = 20 cm = 0.2 m
Source frequency = nth normal mode of frequency, vn = 430 Hz
Speed of sound, v = 340 m/s
In a closed pipe, the nth normal mode of frequency is given by the relation:
v
vn = (2n − 1) 3 n is an integer = 0, 1, 2, 3...
4l

340
430 = (2n − 1)
4 × 0.2

430 × 4 × 0.2
2n – 1 = = 1.01
340
2n = 2.01
n~ 1
Hence, the first mode of vibration frequency is resonantly excited by the given source. In a pipe open
at both ends, the nth mode of vibration frequency is given by the relation:
nv
vn =
2l
2l n n
n=
v
2 × 0.2 × 430
= = 0.5
340
Since the number of the mode of vibration (n) has to be an integer, the given source does not produce
a resonant vibration in an open pipe.
Question 12 : Two sitar strings A and B playing the note ‘Ga’ are slightly out of tune and produce beats of
frequency 6 Hz. The tension in the string A is slightly reduced and the beat frequency is found to reduce to
3 Hz. If the original frequency of A is 324 Hz, what is the frequency of B?
Answer :
Frequency of string A, fA = 324 Hz
Frequency of string B = fB
Beat’s frequency, n = 6 Hz
Beat’s frequency is given as:
n = |fA ± fB|
6 = 324 ± fB
fn = 330 Hz or 318 Hz

31

You might also like